Está en la página 1de 30

BANCO DE PREGUNTAS ONCOLOGÍA La localización más frecuente de los tumores malignos de la mama es:

Cuadrante superior interno


DIAGNÓSTICO Cuadrante superior externo
CASO CLÍNICO 1 Cuadrante inferior interno
Mujer de 65 años de edad, que acude a la consulta con usted por presentar una Cuadrante inferior externo
tumoración no dolorosa en la mama de 3 meses de evolución. Cuenta con Retroareolar
La localización más frecuente de los tumores mamarios es el cuadrante superior externo (50%)
antecedente de consumo de hormonales orales en los últimos 5 años como
El factor pronóstico más importante en cáncer de mama es:
tratamiento para síndrome climatérico, además de consumo de anticonceptivos
Ganglios positivos
orales de los 20 a los 35 años como método de planificación familiar. Tiene 2
HER2 positivo
hijos, a los cuales no les dio lactancia materna. A la exploración física usted
Receptores estrogénicos negativos
detecta tumoración de 2.5cm en cuadrante superior derecho de la mama
Tamaño tumoral
izquierda, con adenopatía ipsilateral de 1cm, móvil. No hay cambios cutáneos
Invasión linfovascular
ni secreción a través del pezón. Todos los anteriores son factores pronósticos en cáncer de mama, sin embargo, el factor pronóstico más
El diagnóstico más probable de esta paciente es: importante hasta ahora es la presencia de ganglios positivos, el cual influye en la supervivencia global (85%
de supervivencia a 5 años en el grupo de ganglios negativos versus 50% en promedio en el grupo de ganglios
Mastopatía fibroquística positivos).
Cáncer de mama CASO CLÍNICO 2
Fibroadenoma mamario Mujer de 40 años, que acude a la consulta por presentar sangrado transvaginal,
Linfoma además de flujo transvaginal fétido. Cuenta con antecedente de haber tenido
Hiperplasia ductal atípica 15 parejas sexuales, algunas de ellas sin uso de métodos anticonceptivos,
Esta paciente tiene factores de riesgo como el uso de hormonales orales, y una tumoración en cuadrante superior
derecho, la localización más común del cáncer de mama, además de adenopatías, por lo que la sospecha además de tener 3 hijos. A la exploración física con tacto vaginal con tumoración
inicial debe ser malignidad. en cérvix, de 4cm, que involucra parametrio derecho. En los estudios de
Su conducta inicial ante esta paciente es la siguiente: extensión se evidencia hidronefrosis derecha con función renal normal.
Iniciar AINEs para desinflamar la glándula ¿Cuál es su diagnóstico presuntivo?
Realizar escisión de la lesión Cervicovaginitis
Mastografía y toma de biopsia Cáncer cervicouterino
Ultrasonido mamario Cáncer de vagina
Iniciar quimioterapia neoadyuvante Quiste de Naboth
Ante la sospecha de malignidad, lo indicado es realizar una mastografía y tomar una biopsia para obtener el
reporte histopatológico.
Cáncer de endometrio
Esta paciente tiene factores de riesgo para cáncer cervicouterino como las múltiples parejas sexuales, la
De acuerdo al tamaño de la lesión, el TNM clínico es: multiparidad. En la exploración clínica tiene una tumoración en cérvix con afección a parametrio, además
T1N0M0 de hidronefrosis, por lo cual la sospecha inicial debe ser cáncer cervicouterino.
T1N1M0 La etapa clínica de acuerdo a la FIGO es:
T2N0M0 IA
T2N1M0 IB
T3N0M0 IIA
Se trata de un tumor de 2.5cm, que corresponde de acuerdo a la clasificación TNM a un T2 (tumor de más de 2 y IIB
hasta 5cm), así como un ganglio ipsilateral móvil, lo cual corresponde a un N1 (ganglio axilar móvil). Por
ahora no hay evidencia de enfermedad metastásica.
IIIB
De acuerdo a la clasificación de la FIGO (2009), la presencia de hidronefrosis clasifica a un paciente en etapa IIIB.
El factor de riesgo más importante es: Para confirmar su diagnóstico usted solicita:
Edad Tomografía de tórax y abdomen
Tabaquismo Marcadores tumorales y ultrasonido testicular
Infección por VPH Radiografía de abdomen
Múltiples parejas sexuales Biometría hemática y urocultivo
Multiparidad Es un diagnóstico clínico
Evidencia señala que esta neoplasia se debe hasta en 98% a la infección por VPH, por lo que se considera como Ante la sospecha de cáncer testicular, los estudios iniciales son el uso de ultrasonido testicular y los marcadores
el factor etiológico. La edad, el tabaquismo, la multiparidad y las múltiples parejas sexuales se consideran tumorales. Posteriormente, la tomografía de tórax y abdomen se utilizan para realizar la etapificación, no
otros factores de riesgo importante. para confirmar el diagnóstico. El diagnóstico definitivo se obtiene mediante el análisis histopatológico de la
Los serotipos de VPH que incluye la vacuna cuadrivalente son: orquiectomía.

6, 11, 16, 18 Los marcadores tumorales útiles en cáncer de testículo son:


16, 18, 31, 33 ACE y DHL
6, 11, 31, 33 Ca19.9, fracción B de la GCH y DHL
16, 17, 18, 20 Alfafetoproteína, fracción B de la GCH y DHL
6, 16, 18, 31 Alfafetoproteína, DHL y Ca125
La vacuna cuadrivalente (Gardasil), protege contra condilomas genitales, displasia y cáncer, al proteger contra los Ca 19.9, Ca 15.3 y DHL
serotipos 6 y 11 (asociados a verrugas vaginales) y 16 y 18 (los más comunes que causan enfermedad Los marcadores tumorales en caso de sospecha de tumor testicular son alfafetoproteína, fracción B de la
maligna). gonadotropina y DHL.
El tratamiento de esta paciente debe ser: Los estudios de extensión confirman enfermedad confinada al testículo. El
Cirugía y radioterapia tratamiento debe ser:
Solo cirugía Vigilancia
Quimiorradioterapia Orquiectomía radical a través del escroto
Quimioterapia paliativa Orquiectomía radical a través de la ingle
Solo radioterapia Quimioterapia
El tratamiento de elección en enfermedad localmente avanzada, que incluye de la etapa IIB, III y IVA, es Antibióticos y reevaluación del paciente
quimiorradioterapia a base de esquemas con platinos, preferentemente cisplatino.
El tratamiento quirúrgico de elección de los pacientes con cáncer testicular es la orquiectomía radical inguinal. El
CASO CLÍNICO 3 uso de quimioterapia está reservado a pacientes con etapas clínicas avanzadas o con riesgo de recurrencia.
Hombre de 25 años que acude con usted por tener aumento de tamaño del Posterior a la cirugía continúa con marcadores tumorales positivos, se realizan
testículo derecho de 3 meses de evolución, no doloroso. Niega otra nuevos estudios que muestran metástasis pulmonares y ganglios
sintomatología. A la exploración física usted detecta testículo derecho retroperitoneales. El tratamiento a seguir es:
aumentado de tamaño, de consistencia dura, no doloroso, transiluminación Quimioterapia
negativa. Sin otros hallazgos relevantes a la exploración física. Radioterapia
Su diagnóstico presuntivo es: Resección de metástasis pulmonares
Hernia inguinoescrotal Vigilancia
Cáncer de testículo Tratamiento paliativo
Varicocele En este caso se trata de un paciente que tiene enfermedad etapa III, ya que tiene metástasis pulmonares, por lo
que el tratamiento de elección de este paciente es quimioterapia. El esquema y número de quimioterapias
Hidrocele dependerá del grupo pronóstico en el que se encuentre este paciente y si es un tumor germinal
Orquiepididimitis seminomatoso o no seminomatoso, pero de forma general debe recibir quimioterapia con intento curativo
Se trata de un hombre con edad de riesgo, con aumento de volumen sin dolor, duro y transiluminación negativa, y con altas posibilidades de éxito.
todos estos hallazgos clínicos son compatibles con cáncer testicular.
CASO CLÍNICO 4 En el caso de cáncer de páncreas, entre 75-90% de los pacientes presentan en el tumor mutación de k-Ras.
También la mutación de BRCA, MSH1 y p53 se han asociado a cáncer de páncreas, aunque en menor
Hombre de 60 años que acude a su consulta por presentar dolor abdominal de proporción, ya que entre todas representan no más del 10% de los casos de cáncer de páncreas.
4 meses de evolución, acompañado de pérdida de peso e ictericia de 3 semanas La localización más frecuente del cáncer de páncreas es:
de evolución. Cuenta con antecedente de tabaquismo intenso por 20 años. A la Cabeza
exploración física destaca ictericia de piel y mucosas, no tiene megalias, pero Cuerpo
usted detecta a la exploración física signo de Courvoisier, dolor a la palpación Cola
en epigastrio, sin palpar alguna masa. Proceso uncinado
Su diagnóstico presuntivo es: No existe una preferencia
Cáncer de vesícula biliar 65-80% de los tumores pancreáticos se localizan en la cabeza del páncreas, 20% en el cuerpo o cola y 5-15% son
difusos.
Cáncer de páncreas
CASO CLÍNICO 5
Pancreatitis crónica
Hombre de 65 años de edad que acude a su consultorio por presentar dolor
Hepatitis aguda
abdominal predominantemente en fosa iliaca derecha, además de
Coledocolitiasis
Se trata de un paciente con factores de riesgo (tabaquismo), además de pérdida de peso e ictericia, lo que debe hematoquezia de 3 meses de evolución, acompañado de pérdida de peso de
hacer sospechar un tumor maligno. La presencia del signo de Courvoisier debe ser un auxiliar para sospechar la 7kg en 3 meses. A la exploración física con palidez de tegumentos, se palpa
presencia de un tumor pancreático.
tumoración en fosa iliaca derecha. Se realiza colonoscopia con lesión exofítica
El signo de Courvoisier es: en colon ascendente de 5cm.
Hepatomegalia dolorosa Su diagnóstico presuntivo es:
Vesícula biliar no dolorosa palpable Enfermedad inflamatoria intestinal
Ganglio periumbilical palpable Cáncer de colon
Ganglio supraclavicular palpable Ameboma
Tromboflebitis migratoria Carcinoide apendicular
El signo de Courvoisier consiste en la palpación de la vesícula biliar de forma indolora y se encuentra presente
hasta en el 25% de los pacientes con cáncer de páncreas. Tuberculosis colónica
El factor de riesgo más importante para la patología que usted considera en Este paciente tiene un cuadro clínico de dolor abdominal, hematoquezia y pérdida de peso, es mayor de 50 años
y se encuentra una lesión exofítica en el colon, por lo que la sospecha diagnóstica debe ser malignidad.
su diagnóstico es: El tipo histológico más frecuente de cáncer de colon es:
Edad Células en anillo de sello
Tabaquismo Adenocarcinoma
Infección por VHC Leiomiosarcoma
Consumo de alcohol GIST
Historia familiar Adenosarcoma
Hasta 30% de los casos de cáncer de páncreas se asocian a consumo de tabaco y se considera el principal factor
Más del 90% de las neoplasias malignas del colon corresponden a adenocarcinomas.
de riesgo. La edad es otro factor de riesgo importante, así como el consumo de alcohol y la historia familiar.
Dentro de las mutaciones genéticas en cáncer de páncreas, la más frecuente es: El marcador tumoral que puede elevarse en estos casos es:
BRCA Alfafetoproteína
K-Ras Ca 19.9
Myc Ca 125
MSH1 ACE
p53 Ca 15.3
El marcador tumoral útil que puede elevarse y es útil en el seguimiento de los pacientes es el ACE, con una Which one is the most frequent subtype of melanoma in Mexico?
sensibilidad del 80% y una especificidad del 90% durante el seguimiento.
Los siguientes son factores de riesgo para cáncer de colon excepto: Superficial spreading melanoma
AINEs Nodular Melanoma
Enfermedad inflamatoria intestinal Lentigo malign melanoma
Síndrome de Lynch Acral lentiginous melanoma
Consumo elevado de carnes rojas Amelanic melanoma
En México, a diferencia de otras regiones del mundo, el melanoma acral lentiginoso es el tipo más común, con
Obesidad más del 50% de los casos.
Se ha observado en varios estudios epidemiológicos que el consumo de AINEs, principalmente aspirina, disminuye The most important prognostic factor in melanoma is:
el riesgo de desarrollar cáncer colorrectal. El resto se considera factores de riesgo.
El escrutinio de cáncer de colon debe empezar a los: Clark level
35 años Breslow level
40 años Lymph nodes with metastases
45 años Size
50 años Ulceration
La presencia de metástasis en los ganglios linfáticos regionales es el factor pronóstico más importante en los
55 años pacientes con melanoma, siendo peor el pronóstico a mayor número de ganglios afectados.
De acuerdo a las guías, el escrutinio de cáncer de colon debe iniciar tanto en hombres como mujeres a partir de Treatment of this patient must include:
los 50 años.
CLINICAL CASE 6 Wide excision and sentinel lymph node biopsy
A 49-year-old man noticed a changing lesion on his right upper back. Over time, Wide excision
the lesion grew darker and raised in appearance. The patient was referred to Wide excision and lymphadenectomy
you. Physical examination showed a 1.5cm skin lesion, with irregular borders Immunotherapy
and color variegation, asymmetrical, with no other abnormalities. No Chemotherapy
El tratamiento de elección en los pacientes que no se encuentran con linfadenopatías al diagnóstico, es la
lymphadenopathy. resección del tumor con márgenes negativos con biopsia de ganglio centinela. Si el ganglio centinela es
Which one is your most likely diagnosis? negativo, no se ha observado beneficio en realizar linfadenectomía en este grupo de pacientes.

Dysplastic nevi CLINICAL CASE 7


Malignant melanoma A 40-year-old female otherwise healthy has noticed a "lump" in her neck, for
Basocelular carcinoma which she seeks evaluation. She has no symptoms. Physical examination reveals
Actinic keratosis an asymmetric thyroid with an approximately 1.5cm nodule on the right side,
Dermatophytosis no lymphadenopathy. She also has thyroid function tests, with normal results.
Se trata de una lesión que ha mostrado crecimiento, además de pigmentación irregular, asimetría y bordes What would be your next step?
irregulares, por lo que la sospecha clínica debe ser melanoma maligno. Perform anti-thyroid antibodies
One of these is not a risk factor for melanoma: Start non-steroidal anti-inflammatory drugs and reassurance
Age Thyroid ultrasound and needle aspiration biopsy
Ultraviolet B rays exposure Start levothyroxine 1.6mcg per kilogram
Smoking Aspiration of cyst
Familial melanoma Ante la presencia de un nódulo tiroideo en un paciente eutiroideo, lo que se recomienda es realizar un ultrasonido
Fair skin con una biopsia por aspiración con aguja fina.
El tabaquismo, a pesar de que puede causar cambios cutáneos y es factor de riesgo para otras neoplasias, no se
ha encontrado como un factor independiente para el desarrollo de melanoma.
Which one is the most frequent histologic type of thyroid cancer? Which one is your most likely diagnosis?
Papillary Gastric cancer
Follicular Esophageal cancer
Medullary Gastro esophageal reflux disease
Anaplastic Peptic ulcer disease
Hürtle cell Pancreatitis
El carcinoma papilar representa del 75-85% de los tumores tiroideos, seguido por el carcinoma folicular (10-15%). Se trata de un paciente con dispepsia, pérdida de peso, dolor abdominal y melena, por lo que la sospecha
The most important prognostic factor in thyroid cancer is: diagnóstica inicial debe ser malignidad. Por la presencia de la sintomatología la sospecha debe ser más
gástrico que esofágico.
Sex What would you do to confirm your presumptive diagnosis?
Lymph nodes with metastases Esophagogastroduodenoscopy and biopsy
Age Abdominal CT
Negative surgical margins Double-contrast barium swallow
Histologic grade PET-CT
El factor pronóstico más importante es la edad, lo pacientes después de los 40-45 años presentan mayor
agresividad local y mayor mortalidad. pHmetry and manometry
Biopsy reports medullary thyroid cancer. Which one of the following genes Ante la sospecha de un tumor gástrico, la endoscopia debe ser el estudio inicial, ya que posibilita la inspección de
la mucosa gástrica y permite la toma de biopsia para confirmar el diagnóstico.
could have a mutation? The most common histological subtype of gastric cancer is:
HER2 Lymphoma
RET Adenocarcinoma- intestinal type
Myc Adenocarcinoma-diffuse type
P53 Leiomyosarcoma
Ras GIST
Las mutaciones en el gen RET se asocian a cáncer medular de tiroides, ya sean mutaciones de novo, o asociadas
Los adenocarcinomas representan el 95% de las neoplasias gástricas, siendo la variante más frecuente la de tipo
a síndromes genéticos como MEN2
intestinal.
Which one of the following syndromes has increased risk of medullary thyroid The most frequent localization of gastric cancer is:
cancer? Cardia
Lynch syndrome Lesser curvature
Multiple endocrine neoplasia type 1 Greater curvature
Multiple endocrine neoplasia type 2 Pyloric area
Hashimoto’s thyroiditis Body and fundus
Familial papillary thyroid cancer La localización más frecuente es el área pilórica, hasta el 50% de los tumores se localizan en esa región, seguida
Entre 20-25% de los tumores medulares de tiroides se asocian a síndromes hereditarios, principalmente la por la curvatura menor (25%), cuerpo y fondo (10%) y cardias (10%).
neoplasia endócrina múltiple tipo 2.
Hereditary gastric cancer has been associated with mutations of:
CLINICAL CASE 8 Her2
A 70-year-old woman presents to your practice with a complaint of dyspepsia Ras
and weight loss. She also refers early satiety and abdominal pain with E-cadherin
nausea and melena. Physical examination with skin pallor, no other physical BRCA 1
findings. VHL
El síndrome de cáncer gástrico hereditario se asocia a mutaciones en el gen de E-cadherina.
CLINICAL CASE 9 One of the following syndromes has been associated with endometrial cancer:
A 70-year-old women with diabetes mellitus and hypertension presents to your Lynch syndrome
practice with abnormal uterine bleeding since 3 months ago. Physical Multiple endocrine neoplasia type 1
examination with BMI 35kg/m2, no other physical findings. Li-Fraumeni syndrome
Which one is your most likely diagnosis? BRCA mutations
Ovarian cancer None of the above
Uterine leiomyomatosis El síndrome de Lynch se encuentra como causa entre el 2-8% de los tumores endometriales, presentándose en
estas pacientes a menor edad.
Cervical cancer CLINICAL CASE 10
Endometrial cancer A 17-year-old male presented with increasing pain in the left upper arm of
Vaginal cancer approximately 3 months' duration and a recent onset of low-grade fever.
Se trata de una mujer postmenopáusica, obesa y diabética con sangrado uterino anormal, por lo que se debe
tener una alta sospecha de malignidad, en este caso, por la sintomatología, la primera sospecha debe ser cáncer On physical examination, there was some local tenderness and soft tissue
de endometrio. swelling over the proximal and mid thirds of the left humerus. Plain
One of these is not a risk factor for endometrial cancer radiograph shows a large ill-defined, destructive, diaphyseal intramedullary
Diabetes mellitus lesion with permeate pattern of bone destruction and periosteal reaction
Smoking of an “onion-peel” type. The lesion is associated with a soft tissue mass.
Obesity Biopsy material showed a highly cellular, infiltrative neoplasm consisting of
Polycystic ovary disease sheets of tightly packed, round cells with very scant cytoplasm ("round blue
Tamoxifen cell tumor").
El tabaquismo se ha asociado con menor riesgo de desarrollar cáncer de endometrio en mujeres
postmenopáusicas, obviamente, el riesgo de salud asociado con el tabaquismo sobrepasa este beneficio. Sin Which one is your most likely diagnosis?
embargo, se ha asociado como reductor a través de estimular el metabolismo hepático de los estrógenos. Los Osteosarcoma
demás son factores de riesgo asociados a cáncer de endometrio.
Ewing sarcoma
Which one is the most frequent histological type of endometrial cancer?
Leiomyosarcoma
Clear cell
Fracture of left humerus
Squamous cell
Bone tuberculosis
Papillary serous El sarcoma de Ewing se presenta con síntomas constitucionales como fiebre de bajo grado, además de apariencia
Endometrioid de capa de cebolla en la radiografía. Dentro de la patología se observa un tumor con células redondas,
pequeñas y azules.
Mucinous
El tipo endometrioide es el más frecuente de cáncer endometrial, representa entre 60-65% Which one of the following translocations has been associated with your
The initial treatment of choice of this patient is: diagnosis?
Chemotherapy 9; 22
Chemotherapy plus radiotherapy 11; 22
Simple hysterectomy 8; 14
Uterine curettage 9; 18
Total abdominal hysterectomy and bilateral salpingo-oophorectomy 14; 18
Del 85-90% de los tumores de Ewing presentan la translocación t (11; 22) (q24; q12) que forma una proteína de
with peritoneal biopsy and cytology fusión anormal denominada EWS-FLI.
El tratamiento quirúrgico del cáncer de endometrio debe incluir histerectomía total abdominal con
salpingooforectomía bilateral, biopsia de epiplón mayor y exploración de la cavidad abdominal.
The most frequent site of metastatic disease in bone sarcomas is: Adenocarcinoma
Liver Carcinoma epidermoide
Lymph nodes Carcinosarcoma
Lung El tipo histológico más frecuente es el carcinoma epidermoide o de células escamosas (80-90%), seguido del
adenocarcinoma y el carcinoma adenoescamoso.
Brain ¿Cuáles son los tipos de VPH a los que se asocia el cáncer cervicouterino?
Soft tissue 6, 11, 16, 18
Hasta el 80% de las metástasis de los sarcomas óseos son al pulmón.
One of these is not a risk factor for bone sarcomas 16, 18, 31, 33
Li-Fraumeni syndrome 6, 16, 26, 31
Irradiated bones 11, 18, 41, 45
Familial cancer 6, 11, 42, 43
En las mujeres mexicanas se han identificado a los tipos de alto riesgo 16 y 18 como los más frecuentes, presentes
Age hasta en el 80% de los casos, los siguientes en frecuencia son 31, 33, 35, 39, 45, 51, 52, 56 y 58.
Mutant p53 ¿Qué estudios de extensión se recomienda realizar?
Aunque la mayor parte de los sarcomas óseos se presentan en pacientes jóvenes, no se ha asociado a la edad
como factor de riesgo para la presencia de estos tumores.
RMN pelvis y ultrasonido hepático
Treatment of this patient must include: Cistoscopia, colonoscopia y colposcopia
Only surgery Rectosigmoidoscopia, urografía excretora, cistoscopia, radiografía de tórax
Only chemotherapy Sólo requiere estadiaje clínico
Chemotherapy and radiotherapy Colposcopia y tomografía del tórax
Dentro de los estudios de extensión que se deben solicitar de acuerdo a la FIGO, se encuentra la
Chemotherapy, radiotherapy and surgery rectosigmoidoscopia, urografía excretora, radiografía de tórax y cistoscopia si se sospecha afección vesical o
There is no standard treatment rectal. Se pueden solicitar TAC y RMN sólo para evaluar la extensión a nivel local.
El tratamiento de los sarcomas de Ewing debe ser multidisciplinario e incluye quimioterapia, generalmente de En los estudios se documenta hidronefrosis bilateral. ¿A qué etapa clínica
forma neoadyuvante, cirugía y radioterapia.
corresponde?
IA
Cáncer en la mujer II
IIIA
CASO CLÍNICO 1
IIIB
Mujer de 45 años de edad, previamente sana. AGO: Menarca a los 13 años, IVS:
IV
15 años, Parejas sexuales: 6; G3P3. Niega consumo de anticonceptivos orales. De acuerdo a la clasificación de la FIGO del 2009, la presencia de hidronefrosis clasifica a la paciente en etapa IIIB.
Pap: nunca. Refiere que inició desde hace 4 meses con sangrado transvaginal ¿Cuál es el tratamiento de esta paciente?
anormal, inicialmente posterior a tener relaciones sexuales y actualmente con Quimioterapia paliativa
sangrado transvaginal cada 15 días. EF: Cardiopulmonar y abdomen sin Quimiorradioterapia definitiva
alteraciones. Tacto vaginal: Se palpa lesión friable en el cérvix, de Cirugía + quimioterapia adyuvante
aproximadamente 6cm, con sangrado en el guante explorador. Resto normal. Cirugía + radioterapia adyuvante
Nefrostomías bilaterales y cuidados paliativos
¿Cuál es el tipo histológico más frecuente del cáncer cervicouterino? El tratamiento de las etapas localmente avanzadas (IIB, III, IVA) es quimiorradioterapia definitiva, principalmente
con esquemas basados en platino.
Células claras
Adenoescamoso
CASO CLÍNICO 2 satisfecha histerectomía total. En el caso de esta paciente que no tiene embarazos previos, la crioterapia o
conización se consideran opciones válidas.
Mujer de 32 años, sin antecedentes relevantes. Acude a centro de salud a La vacuna cuádruple contra VPH protege contra los siguientes serotipos:
realizarse citología cervicovaginal de escrutinio. Refiere haber tenido 2 parejas 16, 18, 31, 33
sexuales, no cuenta con embarazos previos. 6, 11, 16, 18
Los siguientes son factores de riesgo para cáncer cervicouterino, excepto: 6, 16, 26, 31
Antecedente de enfermedad de transmisión sexual 11, 18, 41, 45
Inicio de vida sexual después de los 18 años 6, 11, 42, 43
Múltiples parejas sexuales La vacuna cuadrivalente (Gardasil) protege contra los serotipos 6, 11, 16 y 18, protegiendo contra los condilomas
Tabaquismo y la displasia/cáncer, mientras que la vacuna bivalente (Cervarix) protege contra lo serotipos 16 y 18 (aunque se
ha observado protección cruzada contra los serotipos 31 y 35).
Nunca haberse realizado una citología cervicovaginal
Los factores de riesgo son bajo nivel sociocultural, inicio de vida sexual temprana, múltiples parejas sexuales, CASO CLÍNICO 3
tabaquismo y el hecho de nunca haberse realizado una citología. Mujer de 60 años, previamente sana. Antecedentes ginecoobstétricos: Menarca
De acuerdo a la Norma Oficial Mexicana, se recomienda realizar citología a los 12 años, FUM: 51 años. Nulípara. Acude por presentar desde hace 3 meses
cervicovaginal: una lesión en mama derecha, pétrea, de 3 cm de diámetro. A la exploración
De los 20 a los 64 años, anual física detecta usted además la presencia de un conglomerado ganglionar fijo,
De los 20 a los 64 años, cada 2 años pétreo en la axila ipsilateral. Se realiza biopsia con reporte histopatológico de
De los 25 a los 64 años, anual cáncer de mama, receptores hormonales positivos, Her2 positivo.
De los 25 a los 64 años, cada 2 años El tipo histológico más frecuente de cáncer de mama es:
De los 20 a los 60 años, anual Lobulillar
Si bien existen distintas guías de práctica, tanto nacionales como internacionales, lo que establece la Norma Oficial
Mexicana NOM-014-SSA2-1994, modificada en 2007, es que la citología debe solicitarse a las mujeres entre los 25
Ductal
a los 64 años, en especial con factores de riesgo, de forma anual. Mixto
En esta paciente se reporta citología cervicovaginal con lesión intraepitelial Medular
escamosa de alto grado. La incidencia de NIC3 es aproximadamente: Sarcoma
10% El tipo histológico más frecuente del cáncer de mama es el ductal, que representa hasta el 80% de los casos,
seguido por el lobulillar (10-15%). Las otras variedades histológicas son raras.
20% Los estudios de extensión para descartar enfermedad metastásica que se
30% recomiendan en este caso son:
40% Ninguno porque se trata de una etapa temprana
50% Gammagrama óseo, radiografía de tórax y ultrasonido hepático
La incidencia de NIC 3 en pacientes con lesión intraepitelial de alto grado por citología cervicovaginal es
aproximadamente del 50%. RMN de la mama
El manejo terapéutico de la NIC3 en esta paciente es: Mastografía y ultrasonido mamario
Vigilancia con citología cervicovaginal en 3 meses Tomografía de tórax y abdomen
Vigilancia con citología cervicovaginal en 6 meses En este caso, al haber presencia de un conglomerado ganglionar en la axila, se trata de una etapa localmente
avanzada y está indicada la realización de estudios de extensión. Los estudios indicados de acuerdo a la AJCC son
Colposcopia con toma de biopsia gammagrama óseo, radiografía de tórax y ultrasonido hepático.
Crioterapia o conización Los estudios de extensión no encuentran evidencia de metástasis. De acuerdo
Histerectomía simple al TNM, esta paciente se encuentra en una etapa:
Una vez confirmado el diagnóstico histológico de NIC3, el tratamiento, dado la alta tasa de persistencia o
progresión, si está descartada la invasión y tiene colposcopia satisfactoria, los métodos ablativos o escisionales
T1N0M0
son las terapéuticas aceptadas, entre ellas crioterapia, electrocoagulación, conización; en mujeres con paridad T1N1M0
T2N1M0 De acuerdo a la Norma Oficial Mexicana, se recomienda realizar mastografía:
T2N2M0 De los 40 a los 69 años, anual
T3N2M0 De los 40 a los 69 años, cada 2 años
De acuerdo al TNM, la paciente se encuentra en una etapa T2 (tumor mayor a 2cm y hasta 5cm), N2 De los 50 a los 69 años, anual
(conglomerado axilar ganglionar fijo), M0 (se descartaron metástasis en los estudios de extensión), lo cual
corresponde a una etapa clínica IIIA. De los 50 a los 69 años, cada 2 años
El factor pronóstico más importante en esta paciente es: De los 40 a los 59 años, anual
La NOM-041-SSA2-2011 recomienda realizar mastografía de los 40 a los 69 años, cada 2 años.
Edad
Ganglios positivos Dentro de la mortalidad por cáncer, el cáncer de mama se encuentra en el
Tamaño del tumor lugar:
Receptores hormonales positivos 1
Her2 positivo 2
La presencia de ganglios linfáticos con infiltración maligna es el factor pronóstico más importante en cáncer de 3
mama, siendo la supervivencia global a 5 años 85% en las pacientes con ganglios axilares negativos versus 4
aproximadamente 50% en las pacientes con ganglios positivos (a mayor número de ganglios afectados disminuye
la supervivencia). 5
A partir del 2006, el cáncer de mama superó al cáncer cervicouterino como la primera causa de muerte por cáncer
El medicamento que debe incluir el tratamiento de esta paciente por tener en mujeres mexicanas.
expresión de Her2 es: Los estudios de extensión para descartar enfermedad metastásica que se
Doxorrubicina recomiendan en este caso son:
Trastuzumab Gammagrama óseo, radiografía de tórax y ultrasonido hepático
Bevacizumab RMN de la mama
Rituximab Mastografía y ultrasonido mamario
El tratamiento es el mismo Tomografía de tórax y abdomen
El trastuzumab es un medicamento antiHER2 que ha probado mejorar la supervivencia libre de progresión y la
supervivencia global en las pacientes con sobre expresión de HER2 en cáncer de mama. Ninguno porque se trata de una etapa temprana
En el caso de esta paciente se trata de una etapa temprana cuya posibilidad de enfermedad metastásica es baja,
CASO CLÍNICO 4 por lo que los estudios de extensión no se recomiendan de rutina, a menos que presente síntomas o se encuentren
Mujer de 45 años, sin antecedentes relevantes. Acude por presentar nódulo anormalidades de estudios de laboratorio como elevación de las PFH o de la fosfatasa alcalina.
mamario de 5 meses de evolución que ha aumentado de tamaño de forma El tratamiento inicial de esta paciente debe ser:
progresiva, se agregó recientemente secreción sanguinolenta por el pezón. A la Cirugía conservadora y ganglio centinela
exploración física con nódulo mamario de 1.5cm en cuadrante superior externo, Cirugía conservadora y disección axilar
no se palpan ganglios axilares. Se realiza el diagnóstico de cáncer de mama. Mastectomía y ganglio centinela
Los siguientes son factores de riesgo para cáncer de mama excepto: Mastectomía y disección axilar
Historia familiar de cáncer de mama Quimioterapia
Si no existe contraindicación para recibir radioterapia, enfermedad multicéntrica y dificultad para obtener
Menarca temprana márgenes adecuados o un resultado cosmético aceptable, se puede realizar cirugía conservadora. Dado que la
Menopausia tardía paciente tiene ganglios clínicamente negativos, se puede realizar la exploración del ganglio centinela. En este caso
Lactancia materna la paciente aparentemente tiene sólo un tumor, por lo que se podría realizar cirugía conservadora.

Hiperplasia ductal atípica CASO CLÍNICO 5


Dentro de los factores de riesgo de cáncer de mama, se encuentran la historia familiar de cáncer de mama, la Mujer de 65 años, cuenta con antecedente de diabetes mellitus, hipertensión
menarca temprana, la menopausia tardía y la nuliparidad. Lesiones premalignas como la hiperplasia ductal atípica arterial sistémica en tratamiento. Antecedentes ginecoobstétricos: Menarca a
y el carcinoma ductal in situ se consideran también factores de riesgo. La lactancia materna se considera un factor
protector para el desarrollo de cáncer de mama. los 15 años, G3P3, FUM a los 50 años, con terapia de reemplazo hormonal por
5 años, último Pap hace 2 años negativo a malignidad. Refiere haber iniciado Dentro de los factores pronósticos se encuentran el grado, la invasión al
con sangrado transvaginal hace 3 meses, sin otra sintomatología. EF: IMC 35, miometrio y la etapa clínica
sin otras alteraciones en la exploración física. La mayor parte de las pacientes se presentan con enfermedad confinada al útero. El resto de las aseveraciones
son correctas.
El estudio inicial de esta paciente para confirmar su sospecha diagnóstica es: Se considera la piedra angular del tratamiento en el cáncer de endometrio:
Tomografía de abdomen y pelvis Quimioterapia
Ultrasonido pélvico y toma de biopsia de cavidad endometrial Quimioterapia + radioterapia
Resonancia magnética de pelvis Cirugía
Colposcopía Radioterapia sola
Prueba terapéutica con progestágenos Braquiterapia
En este caso, dado que es una mujer postmenopáusica, obesa, hipertensa y diabética con sangrado uterino
La cirugía es fundamental en el tratamiento ya que evalúa la extensión de la enfermedad, permite la estadificación
anormal, la sospecha debe ser patología endometrial, principalmente cáncer de endometrio. Ante la sospecha, lo
y es terapéutica.
indicado es realizar un ultrasonido para evaluar el grosor del endometrio y una biopsia de la cavidad endometrial
para confirmar el diagnóstico. CASO CLÍNICO 6
El tipo histológico más frecuente de cáncer endometrial es: Mujer de 65 años, previamente sana. Acude por presentar desde hace 5 meses
Seroso papilar pérdida de peso y aumento del perímetro abdominal. A la exploración física se
Endometrioide documenta ascitis, no a tensión. Se realiza ultrasonido abdominal, con hígado
Células claras de bordes regulares, sin alteraciones sonográficas, ascitis y múltiples imágenes
Adenoescamoso compatibles con carcinomatosis peritoneal, además de lesión ovárica izquierda,
Mucinoso sólida, hipervascular, de 6cm de diámetro.
El tipo endometrioide es el más frecuente (60-65%). El carcinoma mucinoso representa menos del 1% de los casos, El tipo histológico más frecuente de cáncer de ovario es:
mientras que el de células claras representa hasta el 4% y el seroso papilar del 1-10% de los casos.
Dentro de los factores de riesgo conocidos para cáncer de endometrio se Tumor germinal
encuentran los siguientes, excepto: Carcinoma seroso papilar
Obesidad Cistadenocarcinoma mucinoso
Diabetes mellitus Carcinoma de células claras
Uso de Tamoxifen Carcinoma epidermoide
Las neoplasias ováricas más frecuentes son las epiteliales, siendo los cistadenocarcinomas serosos los más
Edad menor de 50 años frecuentes (30%), seguidos el endometrioide (10-20%) y el mucinoso (10%). El tumor de células claras representa
Consumo de estrógenos en menopausia del 5-10%, el epidermoide, menos del 1%.
Los factores de riesgo se relacionan con la exposición a estrógenos, como la terapia de reemplazo hormonal, la El marcador tumoral que puede usted solicitar para complementar su
obesidad, nuliparidad. Otros factores de riesgo son la obesidad, la diabetes y la hipertensión. El 75% de casos se sospecha diagnóstica es:
diagnostican en mujeres mayores de 50 años.
Las siguientes afirmaciones sobre el cáncer de endometrio son ciertas, Ca19.9
excepto: Antígeno carcinoembrionario
La mayor parte de las pacientes se presenta con sangrado transvaginal Alfafetoproteína
anormal Ca125
La mayor parte de las pacientes se diagnostica en etapas avanzadas de Ca 15.3
En las neoplasias epiteliales ováricas, uno de los marcadores tumorales de mayor utilidad es el Ca125, que se
metástasis encuentra elevado hasta en el 85% de los tumores ováricos. Sólo en el caso de mujeres jóvenes donde se sospeche
La exploración física en general no muestra datos relevantes la presencia de un tumor germinal de ovario, se debe solicitar alfafetoproteína, fracción β de la gonadotropina y
DHL (menores de 40 años).
La hiperplasia endometrial se considera una lesión premaligna
La siguiente aseveración es cierta con respecto a los marcadores tumorales en Cáncer gastrointestinal
cáncer de ovario: CASO CLÍNICO 1
La elevación del Ca125 es diagnóstica de cáncer de ovario, ya que se eleva hasta Hombre de 65 años, cuenta con antecedente de tabaquismo y alcoholismo
en el 80% de los casos intenso. Refiere que desde hace 4 meses inició con disfagia, pérdida de peso y
La elevación del Ca19.9 es diagnóstica de cáncer de ovario, ya que se eleva hasta dolor retroesternal. Se realizó endoscopia con hallazgo de lesión exofítica
en el 80% de los casos sugestiva de cáncer de esófago.
El Ca125 puede elevarse por causas no malignas que causen irritación El tipo histológico más frecuente de cáncer de esófago es:
peritoneal, en el embarazo y menstruación Adenocarcinoma
El Ca125 no es útil para evaluar respuesta a tratamiento ni para detección de Carcinoma epidermoide
recurrencia Adenoescamoso
No se debe solicitar ningún marcador tumoral en cáncer de ovario Carcinoma con células en anillo de sello
Aunque el Ca125 se eleva hasta en el 85% de los casos, este marcador no es específico de cáncer de ovario y se
encuentra elevado en todas aquellas condiciones no oncológicas que se presentan con inflamación del peritoneo
Linfoma
como pancreatitis, diverticulitis, embarazo, menstruación, así como otras neoplasias como cáncer de mama, Los factores de riesgo para presentar cáncer de esófago son los siguientes,
páncreas, hígado y pulmón. Otra utilidad del Ca125 es evaluar la respuesta a tratamiento, además, es de utilidad excepto:
en la detección temprana de recurrencia.
Dentro de los factores de riesgo para cáncer de ovario se encuentran los Tabaquismo
siguientes, excepto: Alcoholismo
Edad mayor de 50 años Acalasia
Factores genéticos Reflujo gastroesofágico
Menarca temprana y menopausia tardía Infección por Helicobacter pylori
Infertilidad La localización anatómica más frecuente del carcinoma epidermoide es:
Anticonceptivos orales por más de 5 años Tercio superior
La duración de anticonceptivos orales por más de 5 años se considera un factor protector, ya que se ha encontrado Tercio medio
en estudios que tiene un riesgo relativo de 0.58. El resto se considera factores de riesgo para cáncer de ovario. Tercio inferior
El tratamiento inicial del cáncer de ovario debe ser: Unión gastroesofágica
Radioterapia abdominal Se localiza indistinto en todo el esófago
Cirugía con resección de lesión ovárica El síntoma más frecuente del cáncer de esófago es:
Cirugía radical con histerectomía, salpingooforectomía bilateral, omentectomía Pérdida de peso
y biopsias de correderas parietocólicas Disfagia
Quimioterapia Odinofagia
Quimioterapia y radioterapia Reflujo gastroesofágico
En los tumores epiteliales de ovario, la cirugía es muy importante para disminuir la carga tumoral y determinar el
estado clínico. La cirugía ideal es una cirugía conocida como rutina de ovario, donde se realiza lavado en los cuatro Disfonía
cuadrantes y se envía a citología, además de exploración de la cavidad peritoneal, ganglios pélvicos y paraaórticos, El sitio más frecuente de metástasis es:
regiones subdiafragmáticas, histerectomía con salpingooforectomía bilateral, omentectomía, biopsias de
correderas parietocólicas, fondo de saco, peritoneo vesical, ambos hemidiafragmas.
Hígado
Pulmón
Hueso
Pleura
Sistema nervioso central
CASO CLÍNICO 2 Tumor de Krukenberg
Mujer de 45 años de edad, sin antecedentes previos de importancia. Refiere Síndrome de Trousseau
haber iniciado desde hace 4 meses con evacuaciones melénicas, acompañadas CASO CLÍNICO 3
de pérdida de peso de 20 kg en dicho periodo de tiempo. A la exploración Hombre de 56 años, que inició hace 6 meses con pérdida de peso y dolor
física con palidez de tegumentos, además usted detecta adenopatía epigástrico. Se inició manejo con inhibidor de bomba de protones, sin mejoría
supraclavicular izquierda, cardiopulmonar sin alteraciones, abdomen con dolor clínica, por lo que se solicitó un estudio endoscópico. Le reportan un estudio
a la palpación en epigastrio, además de aumento del perímetro abdominal a endoscópico con una lesión ulcerada de bordes elevados localizada en la
expensas de ascitis. región prepilórica. Se toman biopsias con reporte histopatológico de cáncer
El tipo histológico más frecuente de cáncer gástrico es: gástrico.
Adenocarcinoma tipo intestinal De acuerdo al reporte de la endoscopia, corresponde a la clasificación de
Adenocarcinoma tipo difuso Bormann:
Carcinoma epidermoide 1
Tumor del estroma gastrointestinal 2
MALT 3
Los siguientes son factores de riesgo para cáncer gástrico excepto: 4
Consumo de alimentos ahumados 5
Tabaquismo La localización más frecuente del cáncer gástrico es:
Consumo alto de nitratos Unión gastroesofágica
Infección por Helicobacter pylori Curvatura mayor
Dieta rica en vitamina A y C Curvatura menor
Las siguientes son características del carcinoma gástrico intestinal, excepto: Antro y píloro
Se desarrolla a partir de lesiones como gastritis atrófica y metaplasia intestinal Linitis plástica
Es el más frecuente en la mujer Dentro de los estudios de extensión para descartar enfermedad metastásica
Es más frecuente en población de mayor edad se recomienda:
Se asocia con factores como dieta y consumo de tabaco RMN de abdomen y endoscopia
Es frecuente en regiones con cáncer gástrico epidémico TC de tórax y abdomen, ultrasonido transendoscópico
El cáncer gástrico ocupa el siguiente lugar dentro de las causas de muerte Ultrasonido hepático, radiografía de tórax
por cáncer en México: Biometría hemática y antígeno carcinoembrionario
1 No requiere estudios de extensión por el tipo de lesión endoscópica
2 Se confirma que se trata de una lesión con invasión hasta la muscular propia,
3 sin adenopatías regionales o enfermedad metastásica. La mejor conducta
4 terapéutica es la siguiente:
5 Gastrectomía total
La presencia de una adenopatía supraclavicular metastásica se denomina: Gastrectomía parcial
Ganglio de Irish Quimioterapia
Ganglio de la Hermana María José Radioterapia
Ganglio de Virchow Gastroyeyunoanastomosis tipo Billroth I
La forma hereditaria de cáncer gástrico se asocia a mutaciones en: Es más frecuente en mujeres
HER2 Existe predisposición familiar hasta en 8-10% de los casos
E-cadherina La palpación de la vesícula biliar no dolorosa se denomina como:
APC Signo de Trousseau
MLH1 Signo de Courvoisier
BRCA 1 y 2 Síndrome de Lynch
CASO CLÍNICO 4 Triada de Charcot
Hombre de 70 años, antecedente de tabaquismo y consumo de alcohol Signo de Gray-Turner
intenso. Diagnóstico de diabetes mellitus hace 3 meses. Acude por presentar El marcador tumoral útil dentro del abordaje diagnóstico es:
ictericia de 2 semanas de evolución, acompañada de dolor abdominal en Ca125
epigastrio e hipocondrio derecho, además de pérdida de peso de 5kg en 2 Ca19.9
semanas. A la EF usted encuentra ictericia generalizada, cardiopulmonar sin Ca15.3
alteraciones, abdomen con vesícula palpable en hipocondrio derecho, no Antígeno carcinoembrionario
dolorosa. Alfafetoproteína
El tipo histológico más frecuente de cáncer de páncreas es: Las siguientes aseveraciones son ciertas con respecto al Ca19.9 en cáncer de
Adenocarcinoma acinar páncreas, excepto:
Adenocarcinoma ductal La elevación de Ca19.9 se presenta hasta en el 80% de los casos de cáncer de
Carcinoma con células en anillo de sello páncreas
Carcinoma epidermoide La elevación de Ca19.9 se puede presentar en causas benignas como
Cistadenoma mucinoso obstrucción de la vía biliar, hepatitis, pancreatitis y cirrosis hepática
La localización anatómica más frecuente es: Niveles de Ca19.9 > 200UI/ml predicen irresecabilidad de la lesión
Cabeza El Ca19.9 elevado es suficiente para realizar diagnóstico de cáncer de páncreas
Proceso uncinado La elevación de Ca 19.9 se correlaciona con recurrencia durante el
Cuerpo seguimiento
Cola El sitio más frecuente de metástasis es:
Cuello Hueso
Al diagnóstico, lo más frecuente es que se encuentre enfermedad: Hígado
Localizada al páncreas Pulmón
Con invasión vascular Peritoneo
Localmente avanzada Pleura
Metastásica El tratamiento inicial de este paciente es:
Resecable Resección local de la lesión pancreática
Las siguientes aseveraciones son ciertas con respecto al cáncer de páncreas, Pancreatoduodenectomía
excepto: Quimioterapia
El 30% se asocia a consumo de tabaco Radioterapia
La pancreatitis crónica se considera un factor de riesgo Drenaje de la vía biliar
La mutación de k-ras es un suceso inicial en la carcinogénesis La presencia de tromboflebitis migratoria se denomina:
Síndrome de Trousseau Dentro de los marcadores tumorales útiles durante el diagnóstico y
Signo de Gray-Hunter seguimiento se encuentra:
Signo de la Hermana María José Ca125
Signo de Cullen Ca15.3
Síndrome de Cowden Ca19.9
CASO CLÍNICO 5 Antígeno carcinoembrionario
Mujer de 55 años, sin antecedentes relevantes. Inició hace 3 meses con Alfafetoproteína
disminución del calibre de las heces, además de rectorragia en algunas
ocasiones. Al interrogatorio dirigido refiere pérdida de peso de 10kg en 5 Cáncer de piel y vías urinarias
meses de evolución. A la exploración física cardiopulmonar y abdomen sin CASO CLÍNICO 1
alteraciones, tacto rectal se palpa masa de 5 cm en región del recto. Mujer de 70 años, sin antecedentes de importancia. Acude por presentar lesión
Los siguientes son factores de riesgo para carcinoma colorrectal, excepto: cutánea en dorso nasal no dolorosa de varios meses de evolución. A la
Historia familiar de cáncer de colon exploración física usted detecta una lesión con un borde perlado fino elevado
Enfermedad inflamatoria intestinal de 1cm, con algunas telangiectasias, sin evidencia de adenopatías palpables u
Consumo de antiinflamatorios no esteroideos otras alteraciones.
Dieta alta en grasas y baja en fibra El factor de riesgo más frecuente asociado a cáncer de piel es:
Obesidad Radiación UV
Los siguientes síndromes genéticos se asocian a cáncer colorrectal, excepto: Edad
Poliposis adenomatosa familiar Sexo femenino
Síndrome de Cowden Infección por VPH
Síndrome de Peutz-Jeghers Acné
Síndrome de Lynch El tipo histológico más frecuente de cáncer de piel es:
Síndrome de Trousseau Carcinoma espinocelular
La localización anatómica más frecuente es: Carcinoma basocelular
Colon derecho Carcinoma epidermoide
Colon transverso Melanoma maligno
Colon izquierdo Micosis fungoide
Sigmoides La lesión precursora del carcinoma epidermoide de piel es:
Recto Cicatriz previa
La localización más frecuente de enfermedad metastásica en el caso de los Queratosis actínica
pacientes con cáncer de recto es: Léntigo maligno
Hígado Linfedema
Pulmón Papilomatosis
La queratosis actínica es la precursora del carcinoma epidermoide y muestra la misma atipia queratinocítica, daño
Hueso al DNA producida por la radiación UV y mutaciones en p53 producidas por esta.
Peritoneo El tratamiento de elección en el cáncer de piel no melanoma es:
Pleura Cirugía con bordes quirúrgicos negativos
Cirugía con linfadenectomía
Quimioterapia Mitosis
Radioterapia Grado histológico
Vigilancia Presencia de ganglios linfáticos con metástasis
El tratamiento quirúrgico consiste en realizar una extirpación quirúrgica de la lesión con bordes quirúrgicos La presencia de metástasis en los ganglios linfáticos regionales es el factor pronóstico más importante en los
negativos. Dado que la posibilidad de tener enfermedad metastásica es muy baja, no se requiere realizar ningún pacientes con melanoma, siendo peor el pronóstico a mayor número de ganglios afectados.
otro procedimiento. El sitio más frecuente de metástasis es:
La variedad más frecuente de carcinoma basocelular es: Cerebro
Superficial Pulmón
Morfeiforme Tejido celular subcutáneo
Pseudoquístico Ganglios linfáticos
Nodular Hígado
Infiltrativo El sitio más frecuente de metástasis es el tejido celular subcutáneo, seguido de los ganglios linfáticos, el pulmón
La variedad nodular es la más frecuente del carcinoma basocelular, alrededor del 60% corresponden a esta y el hígado.
categoría, y se manifiestan por un nódulo eritematoso traslúcido con telangiectasias. El índice de Breslow se encarga de medir:
CASO CLÍNICO 2 Localización anatómica
Hombre de 60 años, sin otros antecedentes relevantes. Refiere lesión Índice mitótico
hiperpigmentada en región plantar del pie derecho de 5 meses de evolución, la Profundidad de invasión de acuerdo a las capas de la piel
cual ha tenido crecimiento progresivo, con cambios en la pigmentación. A la EF Profundidad de invasión en milímetros
usted encuentra lesión de 2cm, de bordes irregulares, hiperpigmentada, Resultados de Inmunohistoquímica
además de adenopatía inguinal ipsilateral. La medición del grosor de la lesión en milímetros fue descrita en 1970 por Alexandre Breslow, se realiza desde la
Los siguientes factores han sido asociados con el desarrollo de melanoma capa granular de la epidermis hasta el punto más profundo de la invasión.

excepto: CASO CLÍNICO 3


Fenotipo de piel claro Hombre de 65 años, sin antecedentes relevantes. Acude con usted por
Presencia de nevos comunes presentar hematuria macroscópica no dolorosa de 4 meses de evolución. Desde
Historia familiar hace 2 semanas se agregó dolor en flanco derecho. A la exploración física usted
Radiación ultravioleta encuentra una tumoración palpable en flanco derecho de aproximadamente
Tabaquismo 15cm. No adenopatías u otras alteraciones a la exploración física.
El tabaquismo, a pesar de que puede causar cambios cutáneos y es factor de riesgo para otras neoplasias, no se El tipo histológico más frecuente del cáncer renal es:
ha encontrado como un factor independiente para el desarrollo de melanoma. Papilar
La variedad clínico-patológica más frecuente en nuestro país es: Células claras
Melanoma de diseminación superficial Cromófobo
Melanoma lentigo maligno Conductos colectores
Melanoma acral lentiginoso Sarcomatoide
Melanoma nodular El carcinoma de células claras representa del 60-75% de los tumores, seguido por el papilar (15%), cromófobo
Melanoma amelánico (5%).
En México, a diferencia de otras regiones del mundo, el melanoma acral lentiginoso es el tipo más común, con Los siguientes se consideran factores de riesgo para cáncer renal excepto:
más de la mitad de los casos. Obesidad
El factor pronóstico más importante en melanoma es: Falla renal
Índice de profundidad Tabaquismo
Ulceración
Edad El tipo histológico más frecuente de cáncer de próstata es:
Dieta alta en grasas Adenocarcinoma ductal
Si bien el riesgo de un segundo tumor renal se incrementa en los pacientes con diagnóstico a edad temprana, este Adenocarcinoma acinar
riesgo se incrementa debido a que se piensa que puede tener un componente genético más que porque la edad
sea un factor de riesgo. El resto son factores de riesgo conocidos para cáncer renal. Carcinoma epidermoide
Dentro de los síndromes genéticos que se asocian a cáncer renal se encuentra Carcinoma adenoescamoso
el siguiente: Carcinoma con células en anillo de sello
Entre las neoplasias prostáticas malignas, 95% corresponden a adenocarcinomas acinares, y 5% a carcinomas de
Síndrome de Gorlin-Goltz células neuroendocrinas, de células transicionales o sarcomas.
Síndrome de Lynch Los siguientes son factores de riesgo para cáncer de próstata excepto:
Síndrome de von Hippel-Lindau Edad
Neurofibromatosis tipo 1 Raza negra
Neoplasia endócrina múltiple tipo 2 Historia familiar
El síndrome de von Hippel-Lindau está asociado a cáncer renal hereditario y se relaciona con la histología de
células claras, además de quistes renales, pancreáticos, así como hemangioblastomas cerebelosos, angiomas Obesidad
retinianos y feocromocitoma. Ingesta de isoflavonoides
Las siguientes afirmaciones son ciertas con respecto al cáncer renal, excepto: Algunos estudios demuestran cierta acción protectora en sustancias retinoides presentes en frutas y verduras, así
como vitamina E, micronutrimentos, isoflavonoides, selenio y vitamina D.
La mayor parte de los pacientes se encuentran en etapas avanzadas al
diagnóstico Los auxiliares diagnósticos que solicitaría en este caso es:
La triada típica de cáncer renal es dolor, hematuria y masa palpable Examen general de orina, cistoscopia
La triada típica se presenta solo en el 10% de los casos Ultrasonido transrectal con toma de biopsia
Los síndromes paraneoplásicos como hipertensión y anemia son frecuentes en Nueva determinación de APE
esta neoplasia Solo requiere vigilancia
El síndrome de Stauffer es una disfunción hepática que no se vincula con TC de tórax y abdomen
El diagnóstico presuncional se realiza con la determinación de APE y la exploración digital rectal. El ultrasonido
metástasis y es un síndrome paraneoplásico que puede presentarse en estos transrectal permite identificar lesiones sospechosas de malignidad para tomar una biopsia.
pacientes El principal sitio de metástasis es:
La mayor parte de los pacientes se diagnostican en estudios radiográficos que se indican por otra razón, Pulmón
usualmente son tumores pequeños y en etapas iniciales.
El tratamiento en enfermedad localizada es: Ganglios linfáticos
Cirugía Hueso
Quimioterapia Hígado
Quimioembolización Sistema nervioso central
El esqueleto, principalmente el axial, es el principal sitio de metástasis en los pacientes con cáncer de próstata.
Radioterapia Se determina que el paciente tiene una enfermedad localizada, con biopsia
Inhibidores de tirosincinasa con reporte histopatológico Gleason 3+3. La estrategia a seguir con este
Si el tumor está localizado, deberá realizarse resección quirúrgica, siendo el único tratamiento efectivo para el
cáncer localizado. paciente es:
CASO CLÍNICO 4 Quimioterapia
Hombre de 65 años, sin comorbilidades. Acude por presentar disminución del Radioterapia
chorro urinario, pujo y tenesmo, además de nicturia. A la exploración física Resección transuretral de próstata
usted detecta una lesión pétrea en lóbulo prostático derecho en el tacto rectal. Prostatectomía radical
Se realiza un APE con resultado de 8ng/ml. Bloqueo hormonal
Se trata de un paciente con enfermedad localizada, de bajo riesgo por Gleason, por lo que el tratamiento La alfafetoproteína se eleva únicamente en tumores germinales no
dependerá de la esperanza de vida del paciente y si son buenos candidatos quirúrgicos. En este caso, al ser un
hombre sin comorbilidades, se considera un buen candidato quirúrgico, por lo que la prostatectomía radical es el seminomatosos
mejor tratamiento para este paciente. La β-HGC se produce en el sincitiotrofoblasto
CASO CLÍNICO 5 La elevación de marcadores tumorales no es suficiente para realizar el
Hombre de 35 años de edad, previamente sano. Refiere desde hace 5 meses diagnóstico de cáncer testicular aunque el cuadro clínico sea sugestivo
aumento de volumen a nivel testicular, no doloroso, progresivo. Hace 2 meses En los tumores germinales, la alfafetoproteína se restringe a los tumores germinales no seminomatoso.
se agregó pérdida de peso, además de distensión abdominal y dolor. A la El manejo inicial de este paciente debe ser:
exploración física usted documenta adenopatías supraclaviculares, además de Biopsia testicular
testículo derecho con aumento de volumen, no dolor, transiluminación Orquiectomía radical
negativa. Quimioterapia
El factor de riesgo con mayor asociación con cáncer de testículo es: Radioterapia
Síndrome de Klinefelter Hormonoterapia
El tratamiento inicial de un tumor testicular es la orquiectomía radical inguinal, que además brinda un diagnóstico
Infección por VIH definitivo.
Criptorquidia CASO CLÍNICO 6
Edad Hombre de 35 años, sin antecedentes relevantes. Acude por presentar masa
Infertilidad testicular no dolorosa de 1 año de evolución, posteriormente se agregó
El antecedente de criptorquidia es el factor de resigo cuya relación es más clara con el desarrollo de un tumor
testicular, estando presente en el 12% de los casos. lumbalgia desde hace 4 meses y hace 2 semanas con disnea progresiva,
El tumor testicular más frecuente es: acompañada de plétora facial, red venosa colateral en región torácica. A la
Tumor germinal seminomatoso exploración física con edema facial, disfonía, disnea de reposo, red venosa
Tumor germinal no seminomatoso colateral en tórax anterior, abdomen con masa palpable en epi y mesogastrio,
Carcinoma testicular y aumento de volumen a nivel testicular. Marcadores tumorales con β-HGC
Carcinoma epidermoide 200,000, DHL 1530 (<170), AFP 10,490 (0-9). TC con múltiples metástasis
Adenocarcinoma pulmonares, hepáticas, adenopatías mediastinales que comprimen la vena cava
El 60% de los tumores germinales son no seminomatosos, y se dividen en tumores puros (tumores de senos superior, adenopatías retroperitoneales extensas y masa testicular.
endodérmicos, carcinoma embrionario, coriocarcinoma, teratocarcinoma) o mixtos.
¿Qué tipo de tumor presenta el paciente?
Los marcadores tumorales útiles para el diagnóstico son:
Tumor germinal seminomatoso
Alfafetoproteína, antígeno carcinoembrionario y β-HGC
Tumor germinal no seminomatoso
Alfafetoproteína, Ca125 y DHL
Linfoma mediastinal
β-HGC, alfafetoproteína y DHL
Sarcoma retroperitoneal
β-HGC, alfafetoproteína y Ca125
Carcinoma de primario desconocido
β2 microglobulina, alfafetoproteína y DHL En este caso, el paciente tiene marcadores tumorales claramente positivos, por lo que se realiza el diagnóstico de
Los marcadores tumorales que se solicitan en caso de sospecha de tumor testicular son alfafetoproteína, fracción tumor germinal, dado que tiene elevada la alfafetoproteína, el diagnóstico es de un tumor germinal no
β de la gonadotropina y DHL. seminomatoso.
La siguiente aseveración es cierta con respecto a los marcadores tumorales en La sintomatología respiratoria en este paciente está causada por:
cáncer testicular: Tromboembolia pulmonar
La DHL es inespecífica, por lo que no es útil dentro de los marcadores tumorales Metástasis pulmonares
La β-HGC se eleva únicamente en tumores germinales no seminomatosos Síndrome de vena cava superior
Taponamiento cardiaco
Masa abdominal que causa compresión
El síndrome de vena cava superior se caracteriza por disnea progresiva, red venosa colateral, edema facial, plétora
facial, y disfonía y se presenta en tumores germinales.
Su sospecha diagnóstica es:
Las siguientes aseveraciones son ciertas en este caso clínico excepto: Tuberculosis pulmonar
Se requiere forzosamente biopsia testicular para confirmar el diagnóstico Metástasis pulmonares
La presencia del cuadro clínico y los marcadores tumorales elevados justifican Cáncer de pulmón
el inicio de tratamiento Absceso pulmonar
En un paciente con condiciones que pongan en peligro la vida se puede iniciar Tromboembolia pulmonar
quimioterapia antes del tratamiento con orquiectomía En este caso el paciente cuenta con factores de riesgo, además de datos clínicos de síndrome de vena cava
superior y síndrome de Horner, en la radiografía se encuentra una masa apical, por lo que la sospecha debe ser
Las características clínicas corresponden a un tumor de pobre pronóstico
malignidad.
La mayor frecuencia de tumores no seminomatosos es entre los 15-35 años
La presencia del cuadro clínico y los marcadores tumorales elevados justifican el inicio del tratamiento, y dado los
Para confirmar su sospecha diagnóstica usted solicita los siguientes estudios:
marcadores, no es necesario forzosamente realizar una biopsia testicular para confirmar el diagnóstico. BAAR en expectoración y PPD
El siguiente síndrome genético se asocia a tumores germinales primarios TC de tórax y abdomen, con toma de biopsia de la lesión
mediastinales: Broncoscopía con drenaje del absceso
Síndrome de Turner AngioTAC de arterias pulmonares
Síndrome de feminización testicular
Es un diagnóstico clínico
VIH El estudio inicial recomendado es la TC de tórax y abdomen superior, que permite evaluar las características y
Síndrome de Klinefelter extensión del tumor primario, sospechar si existe daño ganglionar mediastinal y puede confirmar o descartar la
Síndrome de Down presencia de metástasis hepáticas o suprarrenales. Posteriormente se realiza una biopsia de la lesión para
El síndrome de Klinefelter se asocia con la presencia de tumores germinales, principalmente de localización confirmar el tipo histológico.
mediastinal. La triada de miosis, ptosis y anhidrosis se denomina:
La edad de presentación más frecuente de los tumores germinales es: Síndrome de Trousseau
Entre los 40-60 años Síndrome de Horner
Entre los 30-50 años Síndrome de Holter
Entre los 10-30 años
Síndrome de Raynaud
Entre los 15-35 años
Síndrome de vena cava superior
Mayores de 60 años El síndrome de Horner se caracteriza por miosis, ptosis y anhidrosis, además de la presencia de enoftalmos. Se da
El mayor pico de incidencia de los tumores germinales es entre los 15 y los 35 años de edad en los hombres.
por afección de la cadena simpática, en este caso por infiltración tumoral.
La presencia de edema facial, red venosa colateral y plétora yugular lo hacen
Misceláneos sospechar de:
CASO CLÍNICO 1 Síndrome de compresión medular
Hombre de 75 años, cuenta con antecedente de tabaquismo intenso desde los Síndrome de Horner
20 años, hasta 2 cajetillas al día. Acude por presentar disnea, plétora yugular y Síndrome de vena cava superior
edema facial. A la exploración física usted encuentra miosis, ptosis y anhidrosis Tamponade cardiaco
de hemicara derecha, además de edema facial y en ambos brazos, red venosa Insuficiencia cardiaca
colateral en tórax anterior. Se solicita una radiografía de tórax donde se observa Todos estos son características clínicas del síndrome de vena cava superior, siendo los tumores más frecuentes
una masa apical izquierda de 6cm que se extiende hasta el mediastino. que lo provocan los linfomas, los tumores germinales y el cáncer de pulmón.
Exposición a radiación
El tipo histológico más frecuente de cáncer de pulmón es: Exposición a humo de leña
Cáncer de células no pequeñas variedad adenocarcinoma Exposición a asbesto
Cáncer de células no pequeñas variedad epidermoide Exposición a radón
Cáncer de células pequeñas El tabaquismo es el principal factor de riesgo de esta neoplasia y se estima que hasta 90% de los casos se deben
al consumo de tabaco.
Cáncer de células grandes
El cáncer de pulmón representa la siguiente causa de muerte por cáncer en
Carcinoma bronquioloalveolar
A lo largo de los últimos años se ha observado una transición en los tipos histológicos del cáncer de pulmón de
hombres:
células no pequeñas, siendo más frecuente la variedad histológica del adenocarcinoma. 1
CASO CLÍNICO 2 2
Mujer de 40 años, cuenta con antecedente de exposición al humo de leña desde 3
la infancia. Acude a la consulta por presentar disnea progresiva, llegando a ser 4
hasta de pequeños esfuerzos, pérdida de peso de 7kg y hace 2 semanas se 5
agregó expectoración con algunos rastros hemáticos. A la exploración física De acuerdo al INEGI, el cáncer de pulmón representa la segunda causa de muerte en hombres por cáncer, sólo
superado por el cáncer de próstata.
usted documenta adenopatías supraclaviculares bilaterales y síndrome de
El tratamiento de una paciente con enfermedad metastásica como este caso
derrame pleural del 50%. Se realiza una TC de tórax y abdomen, donde se
consiste en:
documenta la presencia de una lesión en pulmón derecho, periférica, de 5cm,
Cirugía
acompañada de derrame pleural ipsilateral, adenopatías cervicales,
Quimioterapia paliativa
mediastinales y retroperitoneales, metástasis suprarrenales, hepáticas.
Quimiorradioterapia
Por el cuadro clínico y las características de la paciente, usted piensa que en
Colocación de sonda pleural a permanencia
el reporte histopatológico usted va a obtener el siguiente resultado:
Radioterapia sola
Cáncer de pulmón células no pequeñas variedad adenocarcinoma con El tratamiento de la enfermedad metastásica de forma inicial es quimioterapia paliativa, que ha mostrado mejoría
mutación EGFR negativa en la calidad de vida y beneficio en supervivencia. En este caso asumimos que no se conoce la mutación de EGFR.
De conocerse que está ausente la mutación o que no se puede obtener, el tratamiento debe ser quimioterapia.
Cáncer de pulmón células no pequeñas variedad adenocarcinoma con
En caso de confirmarse mutación de EGFR presente, el tratamiento de esta
mutación EGFR positiva
paciente sería:
Cáncer de pulmón células no pequeñas variedad epidermoide con mutación
Cirugía
EGFR negativa
Quimioterapia paliativa
Cáncer de pulmón células no pequeñas variedad epidermoide con mutación
Inhibidor de tirosincinasa de EGFR
EGFR positiva
Radioterapia
Cáncer de pulmón de células pequeñas
Dentro de las características clínicas para sospechar presencia de mutación EGFR, se encuentra edad menor de 50
Quimiorradioterapia
años, mujeres, sin historia de tabaquismo. En México, otro factor que se ha asociado a presencia de mutación Los inhibidores de EGFR han demostrado mejorar la supervivencia global de los pacientes con mutaciones de
EGFR es la presencia de exposición al humo de leña. EGFR, entre ellos se encuentran erlotinib, gefitinib, afatinib, entre otros, por lo que de conocerse que la mutación
está presente, deben ser la primera línea de tratamiento de estos pacientes.
Dentro de los factores de riesgo en general para cáncer de pulmón, el más
CASO CLÍNICO 3
importante es:
Tabaquismo
Mujer de 40 años, acude por presentar aumento de volumen en región cervical El manejo de esta paciente, si se confirma diagnóstico de malignidad debe ser:
anterior de 5 meses de evolución, progresiva, no dolorosa. A la exploración Administración de I131
física usted palpa aumento del lóbulo tiroideo derecho a expensas de nódulo Cirugía
tiroideo de 3cm, además de adenopatías cervicales ipsilaterales. Usted solicita Quimioterapia
pruebas de función tiroidea, las cuales son normales. Radioterapia
La conducta a seguir es la siguiente: Quimiorradioterapia
Solicitar tiroglobulina El tratamiento óptimo del cáncer de tiroides es la cirugía. La tiroidectomía total o casi total debe realizarse en los
pacientes con cáncer diferenciado con tumores mayores de 1cm, mayores de 45 años, con enfermedad regional.
Solicitar ultrasonido tiroideo y BAAF Si están negativos estos factores de riesgo, y son menores a 1cm sin metástasis, se puede considerar la
Hacer prueba con levotiroxina 100mcg al día por 3 meses y reevaluar hemitiroidectomía.

Vigilancia con nuevas pruebas de función tiroidea en 3 meses CASO CLÍNICO 4


Referir inmediatamente para realizar tiroidectomía Mujer de 44 años, previamente sana. Acude por presentar nódulo en cara
Ante la presencia de un nódulo tiroideo en un paciente eutiroideo, lo que se recomienda es realizar un ultrasonido anterior de cuello de 3cm de 5 meses de evolución. No presenta síntomas. Se
con una biopsia por aspiración con aguja fina.
tomó una biopsia con reporte de neoplasia derivada de células parafoliculares
El tipo histológico más frecuente de cáncer de tiroides es:
(células C).
Papilar
El reporte histopatológico es consistente con:
Folicular
Cáncer papilar de tiroides
Medular
Cáncer folicular de tiroides
Anaplásico
Cáncer medular de tiroides
Linfoma
El carcinoma papilar representa del 75-85% de los tumores tiroideos, seguido por el carcinoma folicular (10-15%).
Cáncer anaplásico de tiroides
El factor pronóstico más importante es: Linfoma tiroideo
El carcinoma medular es una entidad rara derivada de las células C o parafoliculares que se caracteriza por secretar
Metástasis ganglionares calcitonina y corresponde al 5% de las neoplasias tiroideas.
Grado histológico Dentro de los marcadores tumorales útiles en esta neoplasia, se encuentra:
Tamaño del tumor Tiroglobulina
Edad Calcitonina
Metástasis a distancia TSH
El factor pronóstico más importante es la edad, lo pacientes después de los 40-45 años presentan mayor
agresividad local y mayor mortalidad.
Alfafetoproteína
El sitio más frecuente de metástasis es: No existen marcadores tumorales útiles
La evaluación de tumores de tiroides medulares debe incluir medición de calcitonina, que siempre se encuentra
Hueso elevada.
Sistema nervioso central Esta entidad se asocia a mutaciones en el siguiente gen:
Pulmón MET
Tejidos blandos RET
Hígado HER2
Los sitios más frecuentes de metástasis son el pulmón (50%), hueso (25%), sistema nervioso central y otros tejidos
blandos (10%).
EGFR
Transportador de yodo
Mutaciones del gen RET se asocian a cáncer medular de tiroides, ya sean mutaciones de novo o asociadas a Retroperitoneo
síndromes genéticos como MEN2.
Abdomen
El tratamiento de esta paciente debe ser:
Cabeza y cuello
Tiroidectomía total Según su localización, la mayoría se originan en extremidades (59%), tronco (18%), retroperitoneo (13%) y cabeza
Resección únicamente del nódulo tiroideo y cuello (9%).
Quimioterapia El sitio más frecuente de metástasis es:
Radioterapia Hígado
Terapia con I 131 Hueso
El único tratamiento con posibilidad de curación es la cirugía, que es la tiroidectomía total con disección central Cerebro
más disección ipsilateral, si se sospecha enfermedad ganglionar extensa se recomienda que la disección sea
bilateral. Pulmón
Dentro de los casos hereditarios, los más frecuentes son asociados a: Ganglios
La vía hematógena es la forma más común de diseminación a distancia, siendo el pulmón el sitio más afectado
Neoplasia endócrina múltiple tipo 1 (35%), seguido del hígado, hueso y cerebro.
Neoplasia endócrina múltiple tipo 2A Los siguientes son factores de riesgo para sarcomas excepto
Neoplasia endócrina múltiple tipo 2B Síndrome de Gardner
Cáncer de tiroides hereditario Síndrome de Li-Fraumeni
Tiroiditis de Hashimoto Radioterapia
Los pacientes con MEN2A representan 80% de los casos hereditarios, desarrollan liquen plano cutáneo,
amiloidosis, feocromocitoma. Neurofibromatosis tipo 1
CASO CLÍNICO 5 Síndrome de cáncer de mama-ovario
Las mutaciones en BRCA no se han asociado de forma consistente a mayor riesgo de sarcomas.
Hombre de 50 años, acude con usted por presentar aumento de volumen en
El tratamiento inicial de este paciente debe ser:
extremidad inferior derecha, que ha aumentado de forma progresiva, no
Quimioterapia
dolorosa, que limita la deambulación. A la exploración física usted encuentra un
Radioterapia
tumor en muslo derecho de 10cm, duro, fijo a planos profundos, no doloroso.
Quimiorradioterapia
Le realizaron tomografía donde se evidencia lesión dependiente de tejidos
Resección amplia de la lesión
blandos, de 10cm. Se tomó biopsia con reporte histopatológico de sarcoma.
Amputación supracondílea
El tipo histológico más frecuente de sarcoma de extremidades es: El tratamiento primario de los sarcomas de partes blandas es la resección con un margen tridimensional negativo.
Liposarcoma CASO CLÍNICO 6
Leiomiosarcoma
Hombre de 20 años, acude por presentar aumento de volumen en la rodilla
Histiocitoma fibroso maligno
derecha, de 6 meses de evolución, doloroso, acompañado de pérdida de peso
Sarcoma sinovial
de 10kg. A la exploración física, se documenta aumento de volumen en rodilla
Rabdomiosarcoma
En las extremidades, el tipo histológico más frecuente es el histiocitoma fibroso maligno (30%), seguido de
derecha con dificultad para realizar arcos de movimiento en rodilla derecha. Se
liposarcoma y leiomiosarcoma. tomó radiografía de la extremidad con hallazgo de lesión perióstica con signo
La localización más frecuente de los sarcomas de tejidos blandos es: del sol naciente.
Extremidades El tumor óseo más frecuente después del mieloma es:
Tronco Osteosarcoma
Sarcoma de Ewing
Sarcoma de tejidos blandos
Condrosarcoma EXAMEN FINAL
Tumor de células gigantes CASO CLÍNICO 1
Después del mieloma, el osteosarcoma es el tumor óseo primario más frecuente.
Mujer de 65 años, con antecedente de obesidad mórbida. Antecedentes
El sitio anatómico más frecuentemente afectado en los osteosarcomas es:
ginecológicos: Gesta 1, Parto 1; cursó con problemas de fertilidad, requirió uso
Extremo distal del fémur
de hormonales orales. Acude a su consulta por presentar sangrado transvaginal
Extremo proximal del fémur
escaso. A la EF usted no encuentra algo relevante.
Extremo proximal de la tibia
¿Cuál de los siguientes no es un factor de riesgo para neoplasias
Cráneo
endometriales?
Costillas
El osteosarcoma puede afectar a cualquier hueso, pero se localiza preferentemente en las metáfisis de los huesos
Diabetes mellitus
largos, se ubica principalmente en la rodilla, siendo el extremo distal del fémur (40%) la localización más frecuente. Hipertensión arterial
La mutación asociada al sarcoma de Ewing es la siguiente: Obesidad
9; 22 Multiparidad
11; 22 Uso de tamoxifeno
8; 14 Los factores de riesgo para cáncer de endometrio son los relacionados a exposición de estrógenos: obesidad,
nuliparidad, diabetes, hipertensión, antecedentes familiares y antecedente de irradiación pélvica.
9; 18
El estudio que usted indica a su paciente es:
14; 18
Del 85-90% de los tumores de Ewing presentan la translocación t (11; 22) (q24; q12) que forma una proteína de
Ultrasonido transvaginal y toma de biopsia
fusión anormal denominada EWS-FLI. Resonancia magnética de pelvis
El sitio principal de metástasis en los sarcomas óseos es Exploración abdominal
Hígado Tomografía de tórax y abdomen
Ganglios No requiere mayor estudio diagnóstico
Hueso Ante la sospecha de cáncer de endometrio, la exploración pélvica y la especuloscopía pueden mostrar extensión
a cuello o vagina. Sin embargo, ante la sospecha se debe realizar un estudio histopatológico de la cavidad
Pulmón endometrial. El ultrasonido es útil para determinar la presencia de engrosamiento endometrial como abordaje
Tejidos blandos diagnóstico de sangrado uterino anormal.
Hasta el 80% de las metástasis de los sarcomas óseos son al pulmón. La siguiente afirmación es cierta con respecto al cáncer endometrial:
El tratamiento de este paciente debe ser: La presentación más frecuente es dolor pélvico
Multimodal con quimioterapia, cirugía y radioterapia La edad de presentación es en menores de 50 años
Amputación de la extremidad El tipo histológico más frecuente es el carcinoma seroso papilar
Sólo cirugía El 75% se diagnostica en etapas tempranas
Sólo quimioterapia Un endometrio de >1cm puede vigilarse en 6 meses
Sólo radioterapia La presentación clínica más frecuente es sangrado uterino anormal, a menudo en mujeres postmenopáusicas: La
El tratamiento de los sarcomas óseos debe ser multidisciplinario e incluye quimioterapia. edad de presentación es después de la sexta década de la vida. El tipo histológico frecuente es el carcinoma
endometrioide (60-65%). En cualquier mujer que se detecte engrosamiento endometrial >1cm requiere
evaluación y estudio histopatológico para descartar lesiones premalignas o malignas aunque el 75% de los como causa de la sintomatología. Por la presencia de sangrado se deberá descartar la presencia de anemia. Si la
diagnósticos son en etapas tempranas. sospecha es de cáncer colorrectal, la colonoscopia es el estudio de elección, con lo que se puede visualizar
completo el colon y recto, tomar biopsias del tumor y detectar tumores sincrónicos.

El tratamiento inicial de esta paciente es el siguiente: ¿Cuál de los siguientes NO es un factor de riesgo para cáncer de colon?
Legrado uterino Poliposis adenomatosa familiar
Cirugía Enfermedad inflamatoria intestinal
Radioterapia externa Síndrome de Lynch
Braquiterapia Síndrome de Peutz-Jeghers
Quimioterapia Síndrome de Li-Fraumeni
El síndrome de Li-Fraumeni se encuentra asociado a mutaciones en p53 y dentro de los tumores asociados se
La cirugía es útil para evaluar la extensión de la enfermedad y la estadificación además es terapéutica, en las
encuentra cáncer de mama y sarcomas.
pacientes con enfermedad confinada al útero, es la parte inicial y fundamental del tratamiento. En caso de
identificarse factores que indiquen riesgo de recaída se adiciona quimioterapia o radioterapia adyuvante. Se realiza colonoscopia con hallazgo de lesión exofítica de 8 cm a nivel de
Se considera una lesión premaligna endometrial: sigmoides, friable, que ocupa el 80% de la luz. Se toman biopsias con reporte
Hiperplasia endometrial de malignidad. ¿Cuál es el tipo histológico más frecuente?
Endometriosis Tumor del estroma gastrointestinal
Adenomiosis Adenocarcinoma
Endometrioma Carcinoma epitelioide
Miomatosis uterina MALT
La hiperplasia endometrial se ha identificado como una lesión premaligna. Cuando existe atipia, la malignización Linfoma
ocurre entre 8-29% de los casos con hiperplasia simple y compleja, respectivamente.
Hasta 98% de las neoplasias malignas corresponden a adenocarcinomas. Otras neoplasias raras son carcinoide,
sarcoma, linfoma y tumor del estroma gastrointestinal.
CASO CLÍNICO 2 ¿Cuáles son los estudios de extensión que se recomienda realizar en este
Hombre de 55 años, cuenta con antecedente de consumo de tabaco y alcohol paciente?
ocasional. Niega enfermedades crónico-degenerativas. Acude con usted por RMN de abdomen y Ca 19.9
pérdida de peso de 10kg en 3 meses y dolor abdominal en fosa iliaca izquierda, Ultrasonido abdominal y radiografía de tórax
además de notar desde hace 3 meses episodios de rectorragia. A la EF: Palidez TAC de tórax-abdomen y medición de ACE
de tegumentos, SV normales, cardiopulmonar sin compromiso, abdomen no se Endoscopia superior
palpan megalias. Usted realiza tacto rectal sólo con huellas de sangre, no se Gammagrama óseo
palpa tumor. Se recomienda imagen del tórax (Radiografía de tórax o tomografía), tomografía de abdomen y pelvis y
determinación de antígeno carcinoembrionario en sangre. El Ca 19.9 se eleva en otras neoplasias, principalmente
¿Cuál sería su conducta a seguir? pancreatobiliares.
Realizar tomografía de tórax y abdomen como abordaje de pérdida de peso Sus estudios de extensión no muestran enfermedad a distancia. ¿Qué
Iniciar tratamiento sintomático para enfermedad hemorroidal tratamiento requiere este paciente?
Tratamiento con antiespasmódicos y dieta alta en fibra Quimioterapia neoadyuvante
Realizar biometría hemática y colonoscopia Radioterapia abdominal
Pedir coproparasitoscópico y coprocultivo por sospecha de amibiasis Cirugía
En el caso de este paciente con síntomas gastrointestinales y sangrado de tubo digestivo, además de datos de
alarma (pérdida de peso), dado el grupo de edad (>50 años), debe descartarse una neoplasia gastrointestinal
Resección endoscópica de la lesión
Colostomía paliativa Es más frecuente en mujeres
El tratamiento oncológico en el caso de cáncer de colon es multidisciplinario, pero la intervención quirúrgica tiene La mayor parte son resecables al diagnóstico
una participación fundamental en el pronóstico. El tratamiento inicial es la resección quirúrgica, y de acuerdo a
El 85% de los casos se encuentran entre los 60-80 años, con una mediana de 69 años, es más frecuente en el sexo
las características patológicas se decide tratamiento adyuvante, en general con quimioterapia.
masculino con una proporción 1.5:1. 65% de los tumores se ubican en la cabeza del páncreas, 20% en el cuerpo o
CASO CLÍNICO 3 cola y 15% son difusos. Al diagnóstico, sólo 15% de los pacientes son candidatos a cirugía, 40% tiene enfermedad
Hombre de 75 años, cuenta con antecedente de consumo de tabaco y alcohol irresecable y 45% tiene metástasis al diagnóstico. El 85% de los tumores malignos corresponden a
adenocarcinomas ductales.
intenso durante 40 años. Ha presentado ictericia por 6 meses, con coluria y
El cuadro clínico de trombosis venosa profunda asociada a tumor pancreático
acolia, dolor abdominal en epigastrio y pérdida de peso de 10kg en 3 meses.
se denomina:
Acude el día de hoy por aumento de volumen en extremidad inferior izquierda,
Signo de Courvoisier
con dolor, edema y eritema. EF: Conjuntivas ictéricas, ganglio supraclavicular
Signo de Virchow
derecho, abdomen con palpación de la vesícula en hipocondrio derecho, no se
Síndrome de Trousseau
palpa hepato o esplenomegalia. Laboratorios con BT 21 BD 17 BI 4 ALT 55 AST
Síndrome de Lynch
33 FA 455.
Signo de la hermana María José
Su principal sospecha diagnóstica es: El síndrome de Trousseau se refiere a la presencia de trombosis venosa profunda (flebitis migratoria superficial),
Coledocolitiasis descrita en múltiples neoplasias, de las principales el cáncer de páncreas

Hepatitis viral El tratamiento de este paciente es:


Colecistitis aguda Sólo cirugía
Cáncer de páncreas Radioterapia y colocación de prótesis biliar
Pancreatitis crónica Derivación percutánea de la vía biliar
Dentro de los datos clínicos que sugieren la presencia de cáncer de páncreas, se encuentra la edad, el antecedente Cirugía y quimioterapia adyuvante
de tabaquismo, ictericia, pérdida de peso e hiperbilirrubinemia a expensas de bilirrubina directa con síndrome
colestásico. Además, este paciente presenta el signo de Courvoisier (vesícula biliar palpable), asociado con
Quimioterapia y colocación de prótesis biliar
El tratamiento de la enfermedad metastásica es paliativo. No tienen papel el tratamiento quirúrgico o con
neoplasias pancreatobiliares en el 25% de los pacientes.
radioterapia a excepción de medidas paliativas. En este paciente que presenta ictericia obstructiva se debe realizar
El estudio que solicita para confirmar su diagnóstico es: como paliación colocación de prótesis biliar, inicialmente por métodos endoscópicos y solamente de ser fallida
Tomografía trifásica del páncreas y Ca 19.9 valorar colocación de drenajes externos.

Colangiografía pancreática endoscópica CASO CLÍNICO 4


Laparoscopía diagnóstica Mujer de 65 años, no enfermedades previas. Antecedentes ginecoobstétricos:
Perfil de hepatitis viral 1 gesta, 1 parto, FUM hace 4 años. Utilizó anticonceptivos orales por 5 años.
RMN hígado Acude porque se palpó masa en fosa iliaca izquierda, la cual es ligeramente
Para obtener imágenes óptimas en el diagnóstico, se utiliza la TAC trifásica, esto permite distinguir la masa dolorosa. Acudió con otro médico quien le realizó ultrasonido transvaginal con
pancreática y evalúa los vasos peripancreáticos para evaluar resecabilidad. El Ca19.9 se eleva en 80% de los casos.
hallazgo de lesión ovárica izquierda septada, con algunas áreas sólidas,
Se confirma el diagnóstico de cáncer de páncreas, con presencia de metástasis
hipervascular, de 6cm. No se observa líquido libre. Niega otra sintomatología.
hepáticas, peritoneales y ganglionares. La siguiente aseveración es cierta con
¿Cuál es su sospecha diagnóstica?
respecto al cáncer de páncreas:
Enfermedad pélvica inflamatoria
La mayor parte del diagnóstico es en menores de 50 años
Endometriosis
El tipo histológico más frecuente es el adenocarcinoma ductal
Absceso tubo-ovárico
La localización más frecuente es en la cola del páncreas
Cáncer de ovario puede elevar en enfermedades que cursan con inflamación del peritoneo y en otros tumores como cáncer de
mama o pulmonar. 70% del diagnóstico de cáncer de ovario se realiza en etapas avanzadas. Las neoplasias
Trastorno funcional digestivo epiteliales se presentan con mayor frecuencia en la edad adulta, siendo el tipo histológico más frecuente el seroso
Se trata de una paciente mayor de 60 años con masa palpable. Toda mujer con una masa anexial en mujeres papilar (30-40%).
postmenopáusicas, tumor sólido al ultrasonido mayor a 8 cm debe ser sospechosos de malignidad.
De los síndromes hereditarios asociados a cáncer, el más asociado a cáncer de
¿Qué estudios le pueden ayudar a corroborar su sospecha diagnóstica?
ovario es:
Cultivo de secreción cervicovaginal
Trousseau
Prueba terapéutica con anticonceptivos orales
Cowden
Toma de biopsia y determinación de Ca125
Lynch
Prueba terapéutica con antibióticos
BRCA 1 y 2
Colonoscopia
El diagnóstico histológico se realiza mediante biopsia percutánea, abierta o por vía laparoscópica con el fin de
Peutz-Jeghers
Sólo 5-10% de los pacientes con cáncer de ovario son hereditarios. Las mutaciones de BRCA 1 y 2 tienen un riesgo
obtener una muestra adecuada del tumor y realizar una estadificación quirúrgica. No debe realizarse ruptura de
de cáncer de ovario de 15-60%.
la cápsula del tumor por el riesgo de diseminación abdominal. El Ca125 se encuentra elevado hasta en el 85% de
los casos cuando se emplea el límite de corte de 35 UI. CASO CLÍNICO 5
Se confirma su sospecha diagnóstica. El tratamiento inicial de esta paciente Hombre de 25 años, estudiante. No tiene antecedentes de importancia. Inició
debe ser: hace 8 meses con masa testicular no dolorosa, posteriormente refiere lumbalgia
Tratamiento quirúrgico buscando citorreducción óptima (histerectomía, y en los últimos 15 días se ha agregado disnea. A la EF SV normales, no
salpingooforectomía bilateral, omentectomía, biopsia de peritoneo, lavado adenopatías, Abdomen se palpa tumoración de 10cm en mesogastrio, además
peritoneal) se palpa lesión testicular pétrea en testículo izquierdo de 5cm, no dolorosa.
Tratamiento quirúrgico sólo quitando la lesión sospechosa Transiluminación negativa.
Tratamiento quirúrgico con drenaje del absceso ¿Cuál es su sospecha diagnóstica?
Tratamiento antibiótico Varicocele
Quimioterapia Orquiepididimitis crónica
La cirugía es el estándar de oro en la etapificación; el objetivo de la cirugía es retirar toda la enfermedad y Cáncer de testículo
determinar el estadio clínico. La rutina de ovario debe incluir exploración sistemática de la cavidad peritoneal,
histerectomía total y salpingooforectomía bilateral, omentectomía inframesocólica, biopsias de correderas Tuberculosis genital
parietocólicas, fondo de saco y peritoneo visceral, ambos hemidiafragmas y linfadenectomía. Quiste de epidídimo
Una de las siguientes aseveraciones es cierta con respecto al cáncer de ovario: Se trata de un hombre que se encuentra dentro de la edad de riesgo para cáncer testicular, cuya manifestación
más común es una masa escrotal indolora.
El 75% se diagnostica en etapas tempranas
¿Qué estudios solicita para confirmar su sospecha diagnóstica?
El tipo histológico más frecuente es el carcinoma epitelial de células claras
Ultrasonido testicular y marcadores tumorales
El uso de anticonceptivos orales se considera uno de los factores de riesgo
Urocultivo
más fuertes
RMN pelvis
Dentro de los factores de riesgo se encuentran la infertilidad y la
PCR para tuberculosis en orina
menopausia tardía
Se puede establecer clínicamente el diagnóstico
Un Ca125 elevado es diagnóstico de cáncer de ovario
En el caso del abordaje diagnóstico, la ultrasonografía testicular permite distinguir una masa,
Dentro de los factores de riesgo se encuentra la historia familiar, la edad >50 años, menarca temprana,
menopausia tardía, infertilidad. El uso de anticonceptivos orales se considera un factor protector, con un RR 0.58 algunas calcificaciones. Deben cuantificarse siempre los marcadores tumorales. Si no hay
en las que se utilizó por más de 20 años. La elevación del Ca125 es útil, sin embargo, no es diagnóstica, ya que se
confirmación histológica, los marcadores muy elevados son diagnósticos, principalmente en sputum. On review of his symptoms, he reports strenuous fatigue and 15kg
tumores no seminomatosos.
weight loss in the past 3 months. The patient has smoked one pack of cigarettes
El ultrasonido testicular muestra una lesión de 5cm con calcificaciones e
per day for the past 35 years. Physical examination: No lymphadenopathy, chest
hipervascular. Los marcadores tumorales que realiza con DHL 915 (0-150), AFP
with scattered rhonchi bilaterally. No other relevant findings.
75 (0-9), BHGC 255 (0-3). Con estos resultados usted:
Which one is your most likely diagnosis?
Establece el diagnóstico de tumor germinal seminomatoso
Tuberculosis
Establece el diagnóstico de tumor germinal no seminomatoso
Interstitial pneumonia
Requiere esperar a la biopsia testicular para establecer el diagnóstico
Lung cancer
Decide vigilancia estrecha
Community-acquired pneumonia
Repite los estudios de laboratorio
Las características de los estudios de imagen que nos hacen sospechar de una neoplasia testicular son tumores
Chronic obstructive pulmonary disease
Este paciente tiene como factor de riesgo un índice tabáquico elevado, además de síntomas como tos no
sólidos con calcificaciones múltiples y difusas. Dentro de la elevación de los marcadores tumorales, la elevación
productiva, fiebre y posteriormente esputo sanguinolento y pérdida de peso, por lo que dentro de su abordaje de
de alfafetoproteína se restringe a los tumores germinales no seminomatosos.
la hemoptisis deberá descartarse cáncer pulmonar.
Usted realiza una tomografía, donde se observa un conglomerado
What is your next step to confirm your presumptive diagnosis?
retroperitoneal de 5cm, sin metástasis pulmonares o hepáticas. La conducta a
Obtain complete blood count and chest CT
seguir es la siguiente:
Prescribe antibiotics for pneumonia
Iniciar quimioterapia
Mantoux test and bacilloscopy
Administrar radioterapia al retroperitoneo
Gram stain and culture in sputum sample
Realizar orquiectomía radical
Pulmonary-function test and bronchodilators
Tomar biopsia del testículo Dado que la sospecha es cáncer pulmonar, el diagnóstico se debe obtener un estudio de imagen. El estudio inicial
Realizar biopsia de la lesión retroperitoneal recomendado es TC de tórax y abdomen superior, que permite evaluar características y extensión de tumor
El tratamiento inicial y el diagnóstico definitivo se establecen a través de una orquiectomía radical. primario y evaluar si existen ganglios mediastinales y metástasis. En este caso particular, se requiere biometría
Posteriormente, de acuerdo a la etapa clínica, se realizará la elección del tratamiento adyuvante. Sólo los hemática para evaluar si el sangrado tiene repercusión (anemia).
pacientes que se presenten con una urgencia oncológica pueden iniciar con tratamiento con quimioterapia si se In chest CT you find a hilum 7-cm lung mass with ipsilateral pleural effusion
confirma elevación de marcadores tumorales y cuadro clínico sugestivo de tumor germinal.
(30%). What is the best next step?
Posterior al tratamiento que eligió, la siguiente fase consiste en:
Antibiotics and reassurance of the lung mass
Iniciar quimioterapia
Pulmonary function test to evaluate pulmonary reserve to evaluate for
Administrar radioterapia al retroperitoneo
pneumonectomy
Realizar orquiectomía radical
Obtain a specific tissue diagnosis by biopsy of the hilum mass
Tomar biopsia del testículo
Initiate palliative radiation because the patient is not a candidate for curative
Realizar biopsia de la lesión retroperitoneal
Se trata de un paciente con un tumor germinal no seminomatoso con enfermedad retroperitoneal >5cm (Etapa
resection
clínica II). Posterior a la orquiectomía radical estos pacientes requieren quimioterapia basada en platino. Pleurodesis
CLINICAL CASE 6 Es imperativo que cuando se encuentra una masa pulmonar se debe intentar obtener diagnóstico histológico para
determinar el tratamiento. El objetivo primario es determinar el tipo histológico y si el paciente es candidato a
A 68-year old man is brought to the emergency room after coughing up a cupful cirugía, establecer la extensión de la enfermedad.
of red blood. For the previous 3-4 months, he had presented a chronic non- According to tumor location, which histologic type would you expect to find
productive cough, but no fever. More recently, he noticed some blood-streaked in lung biopsy?
Small cell lung cancer positive, PR positive and HER2 negative. According to TNM staging system,
Bronchioloalveolar carcinoma this patient would be:
Adenocarcinoma T1N0M0
Squamous-cell carcinoma T2N0M0
Lymphoma T3N0M0
Los carcinomas escamosos se relacionan más con una localización central, con hombres fumadores y de alta edad, T2N1M0
mientras que los adenocarcinomas se relacionan más con tumores periféricos, en mujeres jóvenes, no fumadores
y con mutación de EGFR. T3N1M0
De acuerdo a la etapificación TNM, esta paciente tiene un tumor de 3cm, lo cual corresponde a la etapa T2
Which one is the leading risk factor for lung cancer? (tumores de 2.1 a 5cm), N0 por no mostrar metástasis axilares clínicas ni confirmadas por patología y no tiene
Alcohol consumption evidencia de enfermedad metastásica (M0), diagnosticándose como etapa clínica IIA, es decir, etapa temprana.
Occupational agents (asbestos) What would be the best initial treatment for this patient?
Radiotherapy Modified radical mastectomy and sentinel node biopsy
Cigarette smoking Breast conserving surgery and axillary dissection
Air pollution Hormonal therapy
Aunque el cáncer de pulmón es una enfermedad multifactorial, el tabaquismo representa el principal factor de Breast conserving surgery and sentinel node biopsy
riesgo para esta neoplasia y hasta 90% de los casos de cáncer de pulmón se deben a consumo de tabaco. Otros
factores de riesgo conocidos son la exposición a asbesto, níquel, arsénico, cadmio y radón, aunque la contribución Modified radical mastectomy and axillary dissection
de estos factores es poco significativa. El tratamiento de la enfermedad temprana (I y II) es quirúrgico, el abordaje incluye la escisión de todo el tejido
involucrado y el manejo de la axila. La cirugía conservadora combinada con radioterapia ofrece los mismos
CLINICAL CASE 7 porcentajes de supervivencia en comparación con mastectomía. En cuanto al manejo de la axila, existen varios
A 57-year-old postmenopausal woman presents for annual examination. She estudios que han mostrado la utilidad del ganglio centinela en mujeres con tumores T1-T2 y ganglios clínicamente
noticed 3 months ago a lump in her right breast. No other symptoms. Physical negativos. Si el ganglio centinela es negativo, no está indicada la disección axilar.

examination: 3cm non-tender, movable, irregular lump, in right breast, no skin The most important prognostic factor in the treatment of this disease is
changes, no nipple discharge. No axillary lymphadenopathy. NO other Age at diagnosis
significant findings. Size of tumor
What will be your next step in this patient? Axillary metastases
Breast ultrasound Estrogen receptors of the tumor
Mammogram HER2 expression of the tumor
El principal factor pronóstico clínico es el estado ganglionar. El pronóstico guarda relación directa con el número
Start non-steroidal anti-inflammatory drugs for 4 weeks and reassurance de ganglios afectados. Otros factores pronósticos son el tamaño de tumor, edad al diagnóstico, invasión
Start hormonal therapy linfovascular, proliferación celular, expresión de receptores hormonales y HER2.

Aspiration of cyst In Mexico, the recommended mammogram screening is:


Ante la presencia de una masa palpable en una paciente postmenopáusica, la mastografía es el mejor método de From 40 to 69 years old, annually
detección para cáncer de mama con sensibilidad de 70-75%. El ultrasonido es un complemento esencial, y es útil
en la identificación de lesiones en mamas densas. Dadas las características de la lesión a la exploración física es
From 40 to 69 years old, every 2 years
importante que el diagnóstico deba establecerse de forma oportuna. From 50 to 69 years old, annually
In mammogram you find a spiculated 3-cm nodule with no lymphadenopathy. From 50 to 69 years old, every 2 years
A biopsy was performed with grade 3 ductal carcinoma of the breast, ER From 40 to 59 years old, every 2 years
De acuerdo a la NOM, se recomienda iniciar escrutinio con mastografía a partir de los 40 años, cada 2 años, hasta
los 69 años de edad.
CLINICAL CASE 8 HPV has been proven to cause LSIL but not atypical squamous cells of
A 26-year-old woman presents to your practice as a new patient for routine undetermined significance (ASCUS) or high-grade squamous intraepithelial
gynecological examination. Her last gynecologic exam was at age 21 when she lesion (HSIL)
was diagnosed with a vaginal yeast infection by the student health department HPV is the virus that causes AIDS, and women with AIDS are 30% more likely
of her university. She reports that other than the yeast infection, her exam and to develop cervical cancer than women without AIDS
Pap were normal. The patient has been sexually active since age 17 and reports There is no known relationship between HPV and cervical cancer
7 partners since then. She uses condoms "about 90%" of the time for pregnancy El virus del papiloma humano tiene un importante papel etiológico en el cáncer cervicouterino. Su asociación con
las lesiones premalignas y malignas del cérvix es igualmente conocida.
and sexually transmitted disease (STD) prevention. She has been in a
What is your next step with this patient with low-grade squamous
monogamous relationship with her current boyfriend for 2 years. She has never
intraepithelial lesion?
been pregnant. She denies any previous diagnosis of an STD, although she has
Repeat Pap test in 12 months
never been tested for HIV. She is having no symptoms at this time.
Repeat Pap test in 6 months
In terms of cervical cancer, what test should be performed today?
Colposcopy
Colposcopy
Perform HPV testing
HPV testing
Immunization against HPV
Pap test Ante el hallazgo de una citología anormal, es indicación de realizar una colposcopia, con el objetivo de revelar
Vaginal examination anormalidades que guíen la toma de biopsias para descartar cáncer invasor.

HIV testing She asks about HPV immunization. According to national immunization
El estudio recomendado para realizar escrutinio de cáncer cervicouterino es la citología cervicovaginal. La schedule in Mexico, what do you recommend?
colposcopia y el examen de VPH son útiles, sin embargo, no están recomendados como estudio inicial de
escrutinio.
You would recommend immunization in this patient because she has low-
In Mexico, the recommended cervical cancer screening is: grade squamous intraepithelial lesion
Pap test from 20 to 64 years annually You would recommend immunization in this patient because she has 26
Pap test from 25 to 64 years annually years old
Pap test since the beginning of sexual intercourse You would not recommend immunization because in Mexico is only
Pap test from 20 to 64 years, every 2 years indicated in 9-year-old females
Pap test from 25 to 64 years, every 2 years You would not recommend immunization because she has low-grade
De acuerdo a la NOM 014 SSA2 1994 con modificaciones en 2007, la edad inicial para la citología cervicovaginal squamous intraepithelial lesion
es a los 25 años, hasta los 64 años de edad. Se puede realizar cada 3 años solo si existen 2 citologías previas There is no immunization against HPV in national immunization schedule in
anuales sin datos de malignidad.
Mexico
You perform a Pap testing and the results show low-grade squamous En México, de acuerdo a la NOM 036 SSA2 2012 se contempla vacunación a los 9 años de edad.
intraepithelial lesion (LSIL). She asks you what the relationship is between CLINICAL CASE 9
HPV and her abnormal Pap. A previously healthy 65-year-old male presents to your practice with 3 months
An abnormal Pap means the cervical tissue is compromised, making it easier with nocturia, difficulty starting and maintaining a steady stream of urine, and
to contract HPV through sexual contact dysuria. No other symptoms. Digital rectal examination with suspicious lump in
HPV infection is a known factor in the vast majority of cervical cancers (and, right side of the prostate. No other physical findings.
therefore, precancerous findings on Pap testing) What will be your next step with this patient?
Urine examination and urine culture Anterior fibromuscular zone
Start treatment with tamsulosin Entre 70-85% de los carcinomas ocurren en la zona periférica, 10-15% en la zona transicional y 10% en la zona
central, hasta 85% son multifocales.
Referral to Urology for transurethral prostatic resection
Which one is not a known risk factor for prostate cancer?
Perform a PSA test and transrectal ultrasound with biopsy
Age
Kegel exercises
Se trata de un hombre > 60 años con síntomas irritativos y obstructivos de próstata, además de una masa
Black ethnicity
sospechosa al tacto rectal. El abordaje diagnóstico se basa en realizar APE y ultrasonido transrectal, el cual permite Alcohol consumption
identificar lesiones sospechosas para guiar la toma de biopsia. Cigarrete smoking
Which one is your most likely diagnosis? Vitamin E consumption
Benign prostatic hyperplasia Los factores de riesgo identificados son edad, raza, historia familiar, tabaquismo, alcoholismo y actividad física.
Prostate cancer Algunos estudios demuestran cierta acción protectora en sustancias retinoides de vegetales o frutas, vitamina E,
selenio y vitamina D.
Prostate abscess
CLINICAL CASE 10
Urinary tract infection
A 65-year-old female arrives to emergency department with gross hematuria.
Chronic prostatitis
De acuerdo al hallazgo de la exploración digital rectal, la sospecha clínica es de cáncer de próstata. La hiperplasia
She had hematuria since 3 months ago, with right-side flank pain and weight
prostática benigna presenta también síntomas irritativos y obstructivos, sin embargo, el tacto rectal se presenta loss. NO other symptoms. Physical examination with pallor, and you detect a
con aumento de volumen sin mostrar lesiones predominantes. Este paciente no cursa con datos clínicos para palpable mass in abdomen.
pensar en causas infecciosas.
Which one is your most likely diagnosis?
A biopsy was performed, with acinar adenocarcinoma of the prostate,
Urinary tract infection
Gleason 3+3. PSA 12ng/ml. Which one would be the best treatment for this
Bladder cancer
patient?
Renal cancer
Radical prostatectomy
Renal glomerulonephritis
External beam radiotherapy
Renal abscess
Watchful waiting Se trata de un hombre de 65 años con hematuria, dolor en flanco y masa palpable, que juntos se conocen como
Transurethral prostatic resection la triada típica de cáncer renal, presente hasta en el 10% de los pacientes. Otras características son fiebre o pérdida
Bilateral orchiectomy de peso.
En términos generales, la enfermedad clínicamente confinada a la próstata se trata primero con cirugía en What would you do to confirm your diagnosis?
pacientes con expectativa de vida mayor a 10 años y buenos candidatos quirúrgicos. SE puede considerar Urine examination and urine culture
observación en un tumor bien o moderadamente diferenciado con esperanza de vida menor a 10 años. La
radioterapia primaria es una buena alternativa en pacientes que rechazan la cirugía y en los que no son buenos Cystoscopy
candidatos quirúrgicos. La RTUP está indicada solo en casos que no pueda realizarse cirugía y requiera Abdominal CT
procedimiento paliativo de obstrucción. La orquiectomía bilateral solo tiene indicación en pacientes con cáncer
de próstata metastásico como tratamiento de depleción hormonal.
Renal biopsy
Which one is the most frequent localization of prostate cancer? Renal ultrasound
La TC confirma el diagnóstico y valora la morfología y función del riñón contralateral, además de la extensión
Central zone extrarrenal y la afección vascular y ganglionar.
Peripheral zone You find an 8-cm renal mass with no invasion to adjacent structures and with
Transitional zone no thrombi, chest x-Ray without metastases, and laboratory test only with
Prostatic urethra anemia, liver function tests normal. The best treatment for this patient is:
Neoadjuvant chemotherapy
Radical nephrectomy
Radiotherapy
Tyrosine-kinase inhibitors
Renal embolization because she has gross hematuria
Si el tumor está localizado debe realizarse nefrectomía radical, que sigue siendo el único tratamiento efectivo para
el cáncer localizado. Se considera un tumor quimiorresistente y radiorresistente. Los inhibidores de tirosin cinasa
como sunitinib tienen papel en enfermedad metastásica.
Which one is not a known risk factor for renal cancer?
Von Hippel Lindau syndrome
Obesity
Cigarette smoking
Chronic renal disease
Chronic urinary infections
Dentro de los factores de riesgo conocidos se encuentra la dieta, la falla renal crónica, tabaquismo, obesidad y
algunos factores relacionados con cáncer renal hereditario como síndrome de von Hippel-Lindau y carcinoma
papilar de células renales.
Which one is the most frequent histological type of renal cancer?
Sarcoma
Clear-cell carcinoma
Transitional carcinoma
Papillary carcinoma
Cromophobe carcinoma
La clasificación de la UICC describe 5 subtipos: carcinoma de células claras (60-75%), papilar (15%), cromófobo
(5%), no clasificables (5%).

También podría gustarte